You are on page 1of 49

9

6
9


r
e
v
i
s
i
o
n
:
2
0
1
1
-
0
5
-
2
8







m
o
d
i
f
i
e
d
:
2
0
1
1
-
0
5
-
2
9


BOREL CONJECTURE AND DUAL BOREL CONJECTURE 1
MARTIN GOLDSTERN, JAKOB KELLNER, SAHARON SHELAH, AND WOLFGANG WOHOFSKY
2
Aasraxcr. We show that it is consistent that the Borel Conjecture and the dual Borel Conjecture hold simulta-
neously.
Ixraootcrrox 3
History. A set X of reals
1
is called strong measure zero (smz), if for all functions f : there are 4
intervals I
n
of measure 1/ f (n) covering X. Obviously, a smz set is a null set (i.e., has Lebesgue measure 5
zero), and it is easy to see that the family of smz sets forms a -ideal and that perfect sets (and therefore 6
uncountable Borel or analytic sets) are not smz.
7
At the beginning of the 20th century, Borel [Bor19, p. 123] conjectured:
Every smz set is countable.
This statement is known as the Borel Conjecture (BC). In the 1970s it was proved that BC is independent, 8
i.e., neither provable nor refutable.
9
Let us very briey comment on the notion of independence: A sentence is called independent of a set 10
T of axioms, if neither nor follows from T. (As a trivial example, (x)(y)x y = y x is independent 11
fromthe group axioms.) The set theoretic (rst order) axiomsystemZFC(Zermelo Fraenkel with the axiom 12
of choice) is considered to be the standard axiomatization of all of mathematics: A mathematical proof is 13
generally accepted as valid i it can be formalized in ZFC. Therefore we just say is independent if 14
is independent of ZFC. Several mathematical statements are independent, the earliest and most prominent 15
example is Hilberts rst problem, the Continuum Hypothesis (CH).
16
BC is independent as well: Sierpi nski [Sie28] showed that CH implies BC (and, since G odel showed 17
the consistency of CH, this gives us the consistency of BC). Using the method of forcing, Laver [Lav76] 18
showed that BC is consistent. 19
Galvin, Mycielski and Solovay [GMS73] proved the following conjecture of Prikry:
X 2

is smz if every comeager (dense G

) set contains a translate of X.


Prikry also dened the following dual notion:
X 2

is called strongly meager (sm) if every set of Lebesgue measure 1 contains


a translate of X.
The dual Borel Conjecture (dBC) states:
Every sm set is countable.
Prikry noted that CH implies dBC and conjectured dBC to be consistent (and therefore independent), 20
which was later proved by Carlson [Car93].
21
Numerous additional results regarding BC and dBC have been proved: The consistency of variants of 22
BC or of dBC, the consistency of BC or dBC together with certain assumptions on cardinal characteristics, 23
Date: 2011-05-28.
2000 Mathematics Subject Classication. Primary 03E35; secondary 03E17, 28E15.
We gratefully acknowledge the following partial support: US National Science Foundation Grant No. 0600940 (all authors);
US-Israel Binational Science Foundation grant 2006108 (third author); FWF Austrian Science Fund grant P21651 and EU FP7 Marie
Curie grant PERG02-GA-2207-224747 (second and fourth author); FWF grant P21968 (rst and fourth author);

OAW Doc fellowship
(fourth author). This is publication 969 of the third author.
1
In this paper, we use 2

as the set of reals. ( = 0, 1, 2, . . ..) By well-known results both the denition and the theorem also
works for the unit interval [0, 1] or the torus R/Z. Occasionally we also write x is a real for x

.
1
9
6
9


r
e
v
i
s
i
o
n
:
2
0
1
1
-
0
5
-
2
8







m
o
d
i
f
i
e
d
:
2
0
1
1
-
0
5
-
2
9


2 MARTIN GOLDSTERN, JAKOB KELLNER, SAHARON SHELAH, AND WOLFGANG WOHOFSKY
etc. See [BJ95, Ch. 8] for several of these results. In this paper, we prove the consistency (and therefore 1
independence) of BC+dBC (i.e., consistently BC and dBC hold simultaneously).
2
The problem. The obvious rst attempt to force BC+dBC is to somehow combine Lavers and Carlsons 3
constructions. However, there are strong obstacles:
4
Lavers construction is a countable support iteration of Laver forcing. The crucial points are:
5
Adding Laver real makes every old uncountable set X non-smz.
6
And this set X remains non-smz after another forcing P, provided that P has the Laver property.
7
So we can start with CH and use a countable support iteration of Laver forcing of length
2
. In the nal 8
model, every set X of reals of size
1
already appeared at some stage <
2
of the iteration; the next Laver 9
real makes X non-smz, and the rest of the iteration (as it is a countable support iteration of proper forcings 10
with the Laver property) has the Laver property, and therefore X is still non-smz in the nal model.
11
Carlsons construction on the other hand adds
2
many Cohen reals in a nite support iteration (or 12
equivalently: nite support product). The crucial points are:
13
A Cohen real makes every old uncountable set X non-sm.
14
And this set X remains non-sm after another forcing P, provided that P has precaliber
1
.
15
So we can start with CH, and use more or less the same argument as above: Assume that X appears at 16
<
2
. Then the next Cohen makes X non-sm. It is enough to show that X remains non-sm at all 17
subsequent stages <
2
. This is guaranteed by the fact that a nite support iteration of Cohen reals of 18
size
1
has precaliber
1
.
19
So it is unclear how to combine the two proofs: A Cohen real makes all old sets smz, and it is easy 20
to see that whenever we add Cohen reals conally often in an iteration of length, say,
2
, all sets of any 21
intermediate extension will be smz, thus violating BC. So we have to avoid Cohen reals,
2
which also 22
implies that we cannot use nite support limits in our iterations. So we have a problem even if we nd a 23
replacement for Cohen forcing in Carlsons proof that makes all old uncountable sets X non-sm and that 24
does not add Cohen reals: Since we cannot use nite support, it seems hopeless to get precaliber
1
, an 25
essential requirement to keep X non-sm.
26
Note that it is the proofs of BC and dBC that are seemingly irreconcilable; this is not clear for the 27
models. Of course Carlsons model, i.e., the Cohen model, cannot satisfy BC, but it is not clear whether 28
maybe already the Laver model could satisfy dBC. (It is even still open whether a single Laver forcing 29
makes every old uncountable set non-sm.) Actually, Bartoszy nski and Shelah [BS03] proved that the Laver 30
model does satisfy the following weaker variant of dBC (note that the continuum has size
2
in the Laver 31
model):
32
Every sm set has size less than the continuum.
33
In any case, it turns out that one can reconcile Lavers and Carlsons proof, by mixing them generi- 34
cally, resulting in the following theorem:
35
Theorem. If ZFC is consistent, then ZFC+BC+dBC is consistent.
36
We give a rather informal overview of the proof in Section 1.
37
Prerequisites. To understand anything of this paper, the reader
38
should have some experience with nite and countable support iteration, proper forcing,
2
-cc, 39
-closed, etc.
40
should know what a quotient forcing is,
41
should have seen some preservation theorem for proper countable support iteration,
42
should have seen some tree forcings (such as Laver forcing).
43
To understand everything, additionally the following is required:
44
The case A preservation theoremfrom[She98], more specically we build on the proof of [Gol93] 45
(or [GK06]).
46
In particular, some familiarity with the property preservation of randoms is recommended. We 47
will use the fact that random and Laver forcing have this property.
1
2
An iteration that forces dBC without adding Cohen reals was given in [BS10], using non-Cohen oracle-cc.
9
6
9


r
e
v
i
s
i
o
n
:
2
0
1
1
-
0
5
-
2
8







m
o
d
i
f
i
e
d
:
2
0
1
1
-
0
5
-
2
9


BOREL CONJECTURE AND DUAL BOREL CONJECTURE 3
We make some claims about (a rather special case of) ord-transitive models in Section 4.A. The 2
reader can either believe these claims, or check them himself (by some rather straightforward 3
proofs), or look up the proofs (of more general settings) in [She04] or [Kel].
4
From the theory of strong measure zero and strongly meager, we only need the following two results 5
(which are essential for our proofs of BC and dBC, respectively):
6
Pawlikowskis result from [Paw96a] (which we quote as Theorem 1.2), and
7
Theorem 8 of Bartoszy nski and Shelahs [BS10] (which we quote as Lemma 3.2).
8
We do not need any other results of Bartoszy nski and Shelahs paper [BS10]; in particular we do not use the 9
notion of non-Cohen oracle-cc. (Note that the third author claims that our construction is more or less the 10
same as a non-Cohen oracle-cc construction introduced in [She06], and that the extended version presented 11
in [She10] is even closer to our preparatory forcing.)
12
The reader does not have to know the original proofs of Con(BC) and Con(dBC), by Laver and Carlson, 13
respectively.
14
Acknowledgment. We thank Tomek Bartoszy nski for pointing out Pawlikowskis result [Paw96a] to us, 15
and for many interesting discussions on the topic.
16
Notation. Stronger conditions in forcing notions are smaller, i.e., q p means that q is stronger than p.
17
Let P Qbe forcing notions. (As usual, we abuse notation by not distinguishing between the underlying 18
set and the quasiorder on it.)
19
For p
1
, p
2
P we write p
1

P
p
2
for p
1
and p
2
are incompatible. Otherwise we write p
1
j p
2
.
20
q

p means that q forces that p is in the generic lter.


21
P is a subforcing of Q means that the relation
P
is the restriction of
Q
to P.
22
P is an incompatibility-preserving subforcing of Q means that P is a subforcing of Q and that 23
p
1

P
p
2
i p
1

Q
p
2
for all p
1
, p
2
P.
24
Let additionally M be a countable transitive
3
model (of a suciently large subset of ZFC) containing P.
25
P is an M-complete subforcing of Q (or: P
M
Q) means that P is a subforcing of Q and: if 26
A P is in M a maximal antichain, then it is a maximal antichain of Q as well. (Or equivalently: 27
P is an incompatibility preserving subforcing of Q and every predense subset of P in M is predense 28
in Q.) Note that this means that every Q-generic lter G over V induces a P-generic lter over M, 29
namely G
M
G P (i.e., every maximal antichain of P in M meets G P in exactly one point). 30
In particular, we can interpret a P-name in M as a Q-name. More exactly, there is a Q-name

31
such that

[G] = [G
M
] for all Q-generic lters G. We will usually just identify and

.
32
Analogously, if P M and i : P Q is a function, then i is called an M-complete embedding if it 33
preserves (or at least

) and and moreover: If A M is predense in P, then i[A] is predense 34


in Q.
35
Annotated contents. 36
Section 1, p. 4: We give a rather informal overview of the whole construction and proof.
37
Section 2, p. 9: We introduce the family of ultralaver forcing notions and prove some properties.
38
Section 3, p. 19: We introduce the family of Janus forcing notions and prove some properties.
39
Section 4, p. 23: We dene ord-transitive models and mention some basic properties. We dene the 40
almost nite and almost countable support iteration over a model. We show that in many 41
respects they behave like nite and countable support, respectively.
42
Section 5, p. 36: We introduce the preparatory forcing notion R which adds a generic forcing iteration

P.
43
Section 6, p. 44: Putting everything together, we showthat RP

2
forces BC+dBC, i.e., that an uncount- 44
able X is neither smz nor sm. We show this under the assumption X V, and then introduce a 45
factorization of R

P that this assumption does not result in loss of generality.
1
3
We will also use so-called ord-transitive models, as dened in Section 4.A.
9
6
9


r
e
v
i
s
i
o
n
:
2
0
1
1
-
0
5
-
2
8







m
o
d
i
f
i
e
d
:
2
0
1
1
-
0
5
-
2
9


4 MARTIN GOLDSTERN, JAKOB KELLNER, SAHARON SHELAH, AND WOLFGANG WOHOFSKY
1. Tnr ovravrrw or rnr raoor 2
In this section, we give a rather informal overview of (and background for) the proof. The emphasis is 3
on giving the reader some vague understanding (i.e., a warm fuzzy feeling), at the expense of correctness 4
of the claims (we point out some of the most blatant lies).
5
1.A. The general setup. We assume CH in the ground model. We use a -closed
2
-cc preparatory 6
forcing R, which adds a generic alternating iteration (as dened below)

P = (P

, Q

)
<
2
. Moreover, R 7
forces that

P is ccc. The forcing notion to get BC+dBC is the composition R P

2
.
8
We say that

P is an alternating iteration if

P = (P

, Q

)
<
2
is a forcing iteration of length
2
satisfy- 9
ing the following:
10
At every even step , (P

forces that) Q

is an ultralaver forcing (described below).


11
At every odd step , (P

forces that) Q

is a Janus forcing (described below).


12
However, instead of using either a Janus or an ultralaver forcing, we are at any step allowed just to 13
do nothing, i.e., set Q

= .
14
At a limit step , we take partial countable support limits. (This means more or less: P

is a 15
subset of the countable support limit of (P

)
<
and contains
_
<
P

and has some other natural 16


properties.)
17
1.B. Ultralaver forcing. Let

D = (D
s
)
s
< be a system of ultralters. The ultralaver forcing L
D
con- 18
sists of trees p with the following property: For every node s p above the stem the set of immediate 19
successors of s is in D
s
. So this is a -centered variant of Laver forcing. Of course this forcing adds a 20
naturally dened generic real, called ultralaver real.
21
We will basically need two properties of ultralaver forcing: The rst one is preservation of positivity: 22
(1.1) L
D
preserves Lebesgue outer measure positivity of ground model sets.
4
The second one is killing of smz sets:
For every uncountable set X in the ground model, L
D
forces that X is non-smz.
Actually, we should formulate this claimin a stronger form. Let us rst quote a result of Pawlikowski [Paw96a], 23
which is essential for the part of our proof that shows BC:
24
Theorem 1.2. X 2

is smz i X + F is null for every closed null set F. 25


Moreover, for every dense G

set H we can construct (in an absolute way) a closed null set F such that for 26
every X 2

with X + F null there is t 2

with t + X H.
27
So we can actually show the following: 28
(1.3)
We can construct from the ultralaver real in an absolute way a (code for a) closed
null set F such that X +F is (outer Lebesgue measure) positive for every uncountable
ground model set X.
It is an easy exercise to show that Theorem 1.2 implies the following fact.
29
Fact 1.4. Assume that

P = (P

, Q

: <
2
) is an iteration with direct limit P

2
satisfying the following:
30
For conally many <
2
, Q

makes every old uncountable set non-smz.


31
P

2
and even all quotients P

2
/P

preserves Lebesgue outer measure positivity.


32
P

2
preserves
1
and satises the
2
-cc.
33
Then P

2
forces BC.
34
Remark 1.5. It is well-known that both Laver reals and randomreals preserve positivity (see Lemma 2.28). 35
As Laver forcing makes every old uncountable set non-smz, we conclude that a countable support iteration 36
of length
2
of Laver reals, or alternatively, a countable support iteration alternating Laver with random 37
reals, forces BC. The latter iteration also forces the failure of dBC, since the random reals increase the 38
covering number of the null ideal, and every set smaller than this cardinal is sm.
1
4
This is a lie, and moreover a stupid (i.e., useless) lie. It is a lie, since we only get something like: for one random over a specic
model, we can nd a system

D such that L
D
preserves randomness. It is a useless lie, since preservation of positivity is not enough
anyway: We need a stronger property that is preserved under proper countable support iterations.
9
6
9


r
e
v
i
s
i
o
n
:
2
0
1
1
-
0
5
-
2
8







m
o
d
i
f
i
e
d
:
2
0
1
1
-
0
5
-
2
9


BOREL CONJECTURE AND DUAL BOREL CONJECTURE 5
1.C. A preparatory forcing for a single step. Let us rst describe how to generically create a single 2
forcing, e.g., an ultralaver forcing.
3
Let Q be a forcing, M a countable transitive model, P M a subforcing of Q. We say that P is an 4
M-complete subforcing of Q, if every maximal antichain A M of P is also a maximal antichain in Q. In 5
this case every Q-generic lter over V induces a P-generic lter over M.
6
Let M
x
be a (countable) model, and

D
x
(D
x
s
)
s
< a system of ultralters in M
x
. This denes the 7
ultralaver forcing Q
x
= L
D
x in M
x
. Given any system

D of ultralters (in V) such that each D
s
extends 8
D
x
s
, then we can show that Q
x
is an M
x
-complete subforcing of the ultralaver forcing Q L
D
in V. We 9
describe this by
5
(M
x
, Q
x
) canonically embeds into Q.
10
So every Q-generic lter H over V induces a Q
x
-generic lter over M
x
which we call H
x
. A trivial but 11
crucial observation is the following: When we evaluate the ultralaver real for Q in V[H] then we get the 12
same real as when we evaluate it for Q
x
in M
x
[H
x
]. Of course Q
x
is not a complete subforcing of Q, just 13
an M
x
-complete one: While Q
x
is just countable, therefore equivalent to Cohen forcing (from the point 14
of view of V), the real added by the Q
x
-generic H
x
is an ultralaver real (over M
x
as well as over V), and 15
therefore does not add a Cohen real over V. 16
We can dene a preparatory forcing R
L
(for a single ultralaver forcing) consisting of pairs x = (M
x
, Q
x
) 17
as above (M in some H(

), say), and ordered as follows: y is stronger than x if M


x
M
y
and (M
y
18
thinks that) (M
x
, Q
x
) canonically embeds into Q
y
. It is not hard to see that R
L
is -closed, and adds in the 19
extension a generic ultralaver forcing Q such that each x in the generic lter embeds into Q.
20
Let G be R
L
-generic (over V). So in V[G], we know that Q
x
is an M
x
-complete subforcing of Q for all 21
x G. Let H be Q-generic (over V[G]). Then H induces a Q
x
-generic lter over M
x
(which we call H
x
) 22
for all x G. Let us repeat the trivial observation: As above, each M
x
[H
x
] will see the real ultralaver 23
real (i.e., the one of V[G][H]).
24
Note that canonical embedding is a form of approximation: If x is in the generic lter, we do not 25
know everything about Q, but we know that Q
x
Q and that the maximal antichains of Q
x
in M
x
will be 26
maximal antichains in Q as well.
27
We should at this stage mention another simple concept that will be used several times: Given (M
x
, Q
x
) 28
and (in V) some ultralaver forcing Q such that (M
x
, Q
x
) embeds into Q, we can take some countable ele- 29
mentary submodel N of H(

) containing (M
x
, Q
x
) and Q, and Mostowski-collapse (N, Q) to y = (M
y
, Q
y
). 30
Then y is in R
L
and stronger than x.
31
1.D. Janus forcing. With Janus forcings we denote a family of forcing notions (as in the case of ul- 32
tralaver forcings). Every Janus forcing J is a subset of H(
1
) and has a countable core (which is the 33
same for every Janus forcing) and some additional stung.
6
The forcing will add a generic real (Janus 34
real) coding a null set Z

. The forcing will not be a complete subforcing of J, but we will require that all 35
maximal antichains involved in the name Z

are also maximal in J, so that J also adds a generic null set Z

.
36
The crucial combinatorial content of Janus forcings heavily relies on previous work by Bartoszy nski 37
and Shelah [BS10].
38
Analogously to the case of ultralaver forcing, let R
J
consist of pairs x = (M
x
, Q
x
) such that M
x
is a 39
countable model and Q
x
a Janus forcing in M
x
. Given x R
J
and a Janus forcing Q in V, we say that x 40
canonically embeds into Q if Q
x
is an M
x
-complete subforcing of Q, and we set y x in R
J
if M
y
thinks 41
that x canonically embeds into Q
y
. Again R
J
is a -closed forcing and adds a generic object Q that is 42
(forced to be) a Janus forcing; and for every x in the R
J
-generic lter, x canonically embeds into Q.
43
As in the case of ultralaver forcing, the Janus real Z

is absolute.
44
Other than in the case of ultralaver forcing, every Janus forcing Q
x
in any model M
x
is itself a Janus 45
forcing in V; so (taking the collapse of an elementary submodel as above) we trivially get: 1
(1.6)
For every x R
J
there is a y x such that in M
y
, Q
y
is a countable Janus forcing (so
in particular equivalent to Cohen forcing).
5
Note the linguistic asymmetry here: A symmetric and more verbose variant would say (M
x
, Q
x
) canonically embeds into
(V, Q).
6
Actually, the denition of Janus forcing additionally depends on a real parameter. In our application, we will use ultralaver
forcings as even stages , and use a Janus forcing dened from the ultralaver real in the stage +1. The following claims about Janus
forcings only hold for this situation; in particular the ground-model sets mentioned have to live in the model before the ultralaver
forcing.
9
6
9


r
e
v
i
s
i
o
n
:
2
0
1
1
-
0
5
-
2
8







m
o
d
i
f
i
e
d
:
2
0
1
1
-
0
5
-
2
9


6 MARTIN GOLDSTERN, JAKOB KELLNER, SAHARON SHELAH, AND WOLFGANG WOHOFSKY

2
R

P V[G] (the generic forcing iteration, direct limit of G)

P
y
M
y

P
x
M
x
M
x
-complete
M
y
-complete
y < x in G
x G
Frotar 1. G is the generic lter for the preparatory forcing notion R, which adds the
generic iteration

P. The forcing that gives BC+dBC is R P

2
. (Of course, in contrast
to the impression given by the diagram, the set M
x

2
, which is the domain of

P
x
, is
not an interval.)
A crucial property of Janus forcing is that we can make it into random forcing as well: 2
(1.7)
For every x R
J
there is a y x such that in M
y
, Q
y
is forcing equivalent to random
forcing.
So here we see an important property of the preparatory forcing R
J
, which might seem a bit paradoxical 3
at rst: Densely, Q seems to be Cohen as well as random forcing. This two-faced behavior gives Janus 4
forcing its name; one could also describe this behavior as faking (faking to be Cohen and faking to be 5
random).
6
Janus forcing is the forcing notion that replaces the Cohen real in the dBC part of the proof. The crucial
point is:
A countable Janus forcing makes every uncountable ground model set of reals non-
sm.
Well, that is actually not much of a point at all: As Carlson has shown, this is achieved by a Cohen real, 7
and obviously a countable Janus forcing is equivalent to a Cohen real. And Carlson even showed: When 8
adding a Cohen real, this uncountable ground model set remains non-sm even after forcing with another 9
forcing notion, provided this forcing notion has precaliber
1
.
10
So what we actually claim for Janus forcing is the more explicit version of our trivial (after Carlson) 11
claim above. First, let us introduce an obvious notation: 12
(1.8)
Let Z be a null set. We say that Z witnesses that X is not sm if there is no real t with
(X + t) Z = , or equivalently, if X + Z = 2

.
(Clearly X is not sm i there is a witness Z.)
So what we really claim is the following: 13
(1.9)
The canonical null set Z

added by a countable Janus forcing has the property that


X + Z

= 2

for all uncountable ground model sets X, and moreover X + Z

= 2

is
preserved by every subsequent -centered forcing.
Of course Z

is interpreted as a code for a null set, not a concrete subset of the reals (otherwise X+Z

= 2

14
could not hold when we add new reals).
15
So the point here is that we can construct the null set Z

(rather: the code) in an absolute way from the 16


Janus real (and not just in a non-canonical way via some equivalence to Cohen forcing that we get out of 17
countability).
18
1.E. The preparatory forcing for the iteration. The preparatory forcing R that we will use will be similar 19
to R
L
or to R
J
, but instead of approximating a single generic ultralaver or Janus forcing, we approximate 20
the alternating iteration

P mentioned in Section 1.A. So our preparatory forcing R consists of pairs x = 1
9
6
9


r
e
v
i
s
i
o
n
:
2
0
1
1
-
0
5
-
2
8







m
o
d
i
f
i
e
d
:
2
0
1
1
-
0
5
-
2
9


BOREL CONJECTURE AND DUAL BOREL CONJECTURE 7
(M
x
,

P
x
), where M
x
is a countable model
7
(and subset of some xed H(

)) and

P
x
is in M
x
an alternating 2
iteration. 3
Assume that x Rand that

P is (in V) an alternating iteration. Other than in the case of a single ultralaver 4
forcing, we nowcannot formally assume that

P
x
is a subset of

P, but there is a natural construction that tries 5
to give an M
x
-complete embedding of

P
x
into

P. If this construction works, we say that x is canonically 6
embeddable into

P, and in that case we can treat

P
x
as subset of

P. So if x is canonically embeddable 7
into

P, and H is a P

2
-generic lter over V (which of course induces P

-generic lters H

for all
2
), 8
we can get a canonical P
x

-generic lter over M


x
for every
2
M
x
(which we call H
x

).
9
We dene the order in R as above: For x, y R, we dene y to be stronger than x, if M
x
M
y
and M
y
10
thinks that x canonically embeds into

P
y
.
11
Note that while M
x
thinks that

P
x
is an iteration of length
2
, in V (or in M
y
for y x) the real domain 12
of

P
x
is just countable (since it is a subset of M
x
).
13
As promised, one can show that R is -closed and adds a generic alternating iteration

P, and that

P is 14
ccc. The nal limit P

2
is the direct limit of the P

(and thus does not add any new reals in the last stage). 15
The intermediate stages satisfy CH, while P

2
forces 2

0
=
2
. As might be expected by now, each x in 16
the R-generic lter G canonically embeds into

P. We will call the R-generic lter G. (The situation is 17
illustrated in Figure 1.) So if H is P

2
-generic over V[G], then we get canonical P
x

2
-generic lters H
x
18
for all x G; and the real ultralaver (and Janus) reals calculated in V[G][H] are the same as the ones 19
locally calculated in M
x
[H
x
].
20
Given any x R we can construct (in V) an alternating iteration

P such that x embeds into

P and such 21
that

P has either of the following two properties:
22
All Janus forcings are countable, at all stages not in M
x
we do nothing, and all limits P

are 23
almost nite support over x (basically the limit is nite support, but we more or less add the 24
countably many elements of P
x

).
25
All Janus forcings are equivalent to randomforcing, and all limits P

are almost countable support 26


over x (basically we take all conditions in the countable support limit that are x-generic).
27
The point is that these iterations behave more or less like nite (or countable) support iterations; but we 28
can still embed x into them. For example, M
x
could think that

P
x
is a countable support iteration, but we 29
may still choose

P to be an almost nite support iteration.
30
Behave more or less in the same way implies in particular in the rst case that any P

is -centered: 31
We iterate only countably many forcings, since we do nothing outside M
x
; the single forcings are - 32
centered (in the ultralaver case) and even countable in the Janus case, and the (almost) nite support limits 33
preserve -centeredness.
34
In the second case, we get preservation of positivity (with respect to outer Lebesgue measure): ultralaver 35
as well as random forcings preserve positivity, and preservation is preserved by (almost) countable support 36
(proper) iterations.
8
37
As above, we put the iteration

P into a countable elementary submodel; collapse it, and thus get: 38
(1.10)
For all x R there is a y x such that (M
y
thinks that) P
y

2
is -centered and all
Janus forcings are countable.
39
(1.11)
For all x R there is a y x such that (M
y
thinks that) P
y

2
preserves Lebesgue outer
measure positivity.
Let us again note that densely often we use nite support, but we also use countable support densely 40
often. 41
1.F. Why BC holds. We want to show that BC is forced by R P

2
. Let X be the name of a set of reals of 42
size
1
. Since P

2
has length
2
, we can assume
9
that X is in the ground model V. We want to show BC, 43
so we have to show that X is not smz. The following is illustrated by Figure 2.
1
7
Since we are interested in iterations of length
2
, we cannot use transitive models, that can only see ordinals <
1
. Instead, we
use ord-transitive models.
8
Of course, this is not true, rather we need an iterable property such as preservation of random reals over models, etc. We do not
get this stronger property universally, we can just preserve a specic random; so claim (1.11) is a lie, too.
9
Well, we cant. But we can do something similar, as will be explained in section 6.B.
9
6
9


r
e
v
i
s
i
o
n
:
2
0
1
1
-
0
5
-
2
8







m
o
d
i
f
i
e
d
:
2
0
1
1
-
0
5
-
2
9


8 MARTIN GOLDSTERN, JAKOB KELLNER, SAHARON SHELAH, AND WOLFGANG WOHOFSKY
+ 1
2
X
y
+ F pos.
F appears at + 1 Assume
R

P
X + F Z
(3) pick x s.t.:
(4) pick y x s.t:
Z is P
x

-name
(5) contradiction!
so: X
y
+ F pos.
(c) P
y
pres. pos.
(X
y
X,
F, Z abs.)
x
y
R

P
x

P
y

P
(1) (2)
(b) (d)
Frotar 2. The proof of BC.
(1) Fix any ultralaver position . (Well, we x large enough to justify our assumption that X V.) 2
We know that the ultralaver real that is added by Q

(i.e., appears at stage + 1) denes in an 3


absolute way a (code for a) closed null set F.
4
(2) According to Theorem 1.2, it is enough to show that X + F is non-null in the extension by R P

2
5
(where the Borel code F is evaluated in the extension). So assume towards a contradiction that 6
X + F is forced to be a subset of a null set (or rather, a Borel code) Z; this already has to happen
10
7
at some stage <
2
. In other words: We assume (towards a contradiction) 8

X + F Z.
(3) Since P

is (forced to be) ccc, we can nd a very absolute (countable) name for Z; and we can 9
nd an x R that already calculates Z correctly.
11
10
(4) Now we construct (in V) a y x in R (with

P
y

proper) that satises (1.11), and moreover such that 11


X
y
XM
y
M
y
is uncountable in M
y
(we get this for free if M
y
is the collapse of an elementary 12
submodel N with X N) In particular, (M
y
thinks that)
13
(a) P
y

is proper, thus preserves


1
, thus forces that X
y
is uncountable.
14
(b) Therefore Q
y

forces that X
y
+ F is positive (according to (1.3)).
15
(c) P
y

2
preserves positivity.
16
(d) Therefore P
y

forces that X
y
+ F is positive
12
(and in particular not a subset of Z).
17
(5) This leads to the obvious contradiction: Let G be R-generic over V and contain y, and let H

be 18
P

-generic over V[G]. Then H


y

is P
y

-generic over M
y
, and therefore M
y
[H
y

] thinks that some 19


x + f X
y
+ F is not in Z. But X
y
= X M
y
X, and the codes for F and for Z are absolute (for 20
F since it is constructed in a canonical way from the ultralaver real, for Z because we took care of 21
it in step (3)).
22
1.G. Why dBC holds. The proof of dBC is similar, using -centeredness instead of positivity preserving, 23
and a countable Janus forcing instead of ultralaver forcing.
24
Let X be the name of a set of reals of size
1
. Again, without loss of generality
13
X V. We want to 25
show dBC, so we have to show that X is not sm.
26
(1) Fix any Janus position (large enough to justify our assumption that X V). We know that the 27
Janus real that is added by Q

(i.e., appears at stage + 1) denes in an absolute way a (code for 28


a) null set Z

.
1
10
Each real (and in particular the Borel code Z) in the P

2
-extension already has to appear at some stage <
2
; and the statement
X + F Z is absolute.
11
More formally: We nd an x R and a P
x

-name Z
x
in M
x
such that x forces (in R) that P

forces that Z (evaluated by the


P

-generic) is the same as Z


x
(evaluated by the induced P
x

-generic).
12
Here we even get positivity of X
y
+ F where F is evaluated in the intermediate extension of stage + 1. However, we get the
contradiction even if we just assume that X
y
+ F is positive where F is evaluated in the P
y

-extension.
13
And again, this is a lie.
9
6
9


r
e
v
i
s
i
o
n
:
2
0
1
1
-
0
5
-
2
8







m
o
d
i
f
i
e
d
:
2
0
1
1
-
0
5
-
2
9


BOREL CONJECTURE AND DUAL BOREL CONJECTURE 9
(2) According to (1.8), it is enough to show that Z

+ X = 2

in the extension by R P

2
. So assume 2
towards a contradiction that Z

+ X 2

. This already happens at some stage < , i.e., we 3


assume 4

R

P

r Z

+ X.
(3) Again, nd x such that r is an absolute P
x

-name.
5
(4) Now we construct (in V) a y x in R that satises (1.10), and such that X
y
X M
y
M
y
is 6
uncountable in M
y
. In particular, (M
y
thinks that)
7
(a) P
y

is proper, thus preserves


1
, and so forces that X
y
is uncountable.
8
(b) Q
y

is (forced to be) a countable Janus forcing notion.


9
(c) P
y

2
is -centered.
10
(d) Therefore (1.9) implies that P
y

forces that Z

+ X
y
= 2

, in particular that r Z

+ X
y
.
11
(5) As before, this leads to a contradiction.
12
2. Uiraxixvra roacrxo 13
In this section, we dene the family of ultralaver forcings L
D
, variants of Laver forcing which depend 14
on a system

D of ultralters. We will need the following properties of this forcing notion:
15
Wish List 2.1. (1) L
D
is -centered, hence ccc. (This will follow trivially from the denition.)
16
(2) Ultralaver kills smz: There is a canonical L
D
-name

for a fast growing real in

called the 17
ultralaver real. From this real, we can dene a closed null set F such that X + F is positive for all 18
uncountable X in V (and therefore F witnesses that X is not smz, according to Theorem 1.2). (See 19
Corollary 2.20.)
20
(3) Canonical above in particular means: If M is a countable model, L
D
M M is an ultralaver 21
forcing (with ultralaver real

M
) which M-completely embeds into the ultralaver forcing L
D
(with 22
ultralaver real

), then in the L
D
-extension V[H],

[H] is the same as


M
[H
M
], where H
M
is the 23
induced lter on L
D
M , similarly for F.
24
(4) Whenever X is uncountable, then L
D
forces that X is not thin. (See Corollary 2.23.)
25
(5) If (M, ) is a countable model of ZFC* and if L
D
M is an ultralaver forcing in M (with ultralaver 26
real

M
), then for any ultralter system

D extending

D
M
, L
D
M is an M-complete subforcing of the 27
ultralaver forcing L
D
(which adds the same ultralaver real). (See Lemma 2.4.)
28
(6) Moreover, given M and L
D
M as above, and a random real r over M, we can choose

D extending 29

D
M
such that L
D
forces that randomness of r is preserved (in a strong way that can be preserved in 30
a countable support iteration). (See Lemma 2.29.)
31
(Advice to the reader: At rst reading, you may want to take these properties as granted, and skip to 32
Section 3.)
33
2.A. Denition of ultralaver. 34
Notation. We use the following fairly standard notation:
35
A tree is a nonempty set p
<
which is closed under initial segments and has no maximal elements.
14
36
The elements (nodes) of a tree are partially ordered by .
37
For each sequence s
<
we write lh(s) for the length of s.
38
For any tree p
<
and any s p we write succ
p
(s) for one of the following two sets: 39
k : s

k p or t p : (k ) t = s

k
and we rely on the context to help the reader decide which set we mean.
40
A branch of p is either of the following:
41
A function f : with f n p for all n .
42
A maximal chain in the partial order (p, ). (As our trees do not have maximal elements, each 43
such chain C determines a branch
_
C in the rst sense, and conversely.)
1
14
Except for the proof of Lemma 2.4, where we also allow trees with maximal elements, and even empty trees.
9
6
9


r
e
v
i
s
i
o
n
:
2
0
1
1
-
0
5
-
2
8







m
o
d
i
f
i
e
d
:
2
0
1
1
-
0
5
-
2
9


10 MARTIN GOLDSTERN, JAKOB KELLNER, SAHARON SHELAH, AND WOLFGANG WOHOFSKY
We write [p] for the set of all branches of p.
2
For any tree p
<
and any s p we write p
[s]
for the set t p : t s or t s, and we write [s] for 3
either of the following sets: 4
t p : s t or x [p] : s x.
The stem of a tree p is the shortest s p with succ
p
(s) > 1. (The trees we consider will never be 5
branches, i.e., will always have nite stems.)
6
Denition 2.2. For trees q, p we write q p if q p (q is stronger than p), and we say that q 7
is a pure extension of p (q
0
p) if q p and stem(q) = stem(p).
8
A lter system

D is a family (D
s
)
s
< of lters on . (All our lters will contain the Fr echet lter 9
of conite sets.) We write D
+
s
for the collection of D
s
-positive sets (i.e., sets whose complement is 10
not in D
s
).
11
We dene L
D
as the set of all trees p such that succ
p
(t) D
+
t
for all t p above the stem.
12
The generic lter is determined by the generic branch

= (
i
)
i

, called the generic real: 13

=
_
pG
[p] or equivalently,

=
_
pG
stem(p).
14
An ultralter system is a lter system consisting of ultralters. (Since all our lters contain the 15
Fr echet lter, we only consider nonprincipal ultralters.)
16
An ultralaver forcing is a forcing L
D
dened from an ultralter system. The generic real for an 17
ultralaver forcing is also called the ultralaver real.
18
Recall that a forcing notion (P, ) is -centered i P =
_
n
P
n
, where for all n, k and for all 19
p
1
, . . . , p
k
P
n
there is q p
1
, . . . , p
k
.
20
Note that all ultralaver forcings L
D
are -centered (fullling our rst wish 2.1(1)), as nite set of con- 21
ditions with the same stem has a common lower bound. If each D
s
is the Fr echet lter, then L
D
is Laver 22
forcing (often just written L).
23
2.B. M-complete embeddings. Note that for all ultralter systems

D we have: 24
(2.3)
Two conditions in L
D
are compatible if and only if their stems are comparable and
moreover, the longer stem is an element of the condition with the shorter stem.
Lemma 2.4. Let M be countable.
15
In M, let L
D
M be an ultralaver forcing. Let

D be (in V) a lter system 25
extending
16

D
M
. Then L
D
M is an M-complete subforcing of L
D
.
26
Proof. For any tree
17
T, any lter system

E = (E
s
)
s
<, and any s
0
T we dene a sequence (T

E,s
0
)

1
27
of derivatives (where we may abbreviate T

E,s
0
to T

) as follows:
28
T
0
T
[s
0
]
. 29
Given T

, we let T
+1
T

\
_
[s] : s T

, s
0
s, succ
T
(s) E
+
s
, where [s] t : s t.
30
For limit ordinals > 0 we let T

_
<
T

.
31
Then we have 32
(a) Each T

is closed under initial segments. Also: < implies T

.
33
(b) There is an
0
<
1
such that T

0
= T

0
+1
= T

for all >


0
. We write T

or T

E,s
0
for T

0
.
34
(c) If s
0
T

E,s
0
, then T

E,s
0
L
E
with stem s
0
. 35
Conversely, if stem(T) = s
0
, and T L
E
, then T

= T.
36
(d) If T contains a tree q L
E
with stem(q) = s
0
, then T

contains q

= q, so in particular s
0
T

.
37
(e) Thus: T contains a condition in L
E
with stem s
0
i s
0
T

E,s
0
.
38
(f) The computation of T

is absolute between any two models containing T and



E. (In particular, 39
any transitive ZFC*-model containing T will also contain
0
.)
1
15
Here, we can assume that M is a countable transitive model of a suciently large nite subset ZFC* of ZFC. Later, we will also
use ord-transitive models instead of transitive ones, which does not make any dierence as far as properties of L
D
are concerned, as
our arguments take place in transitive parts of such models.
16
I.e., D
M
s
D
s
for all s
<
.
17
Here we also allow empty trees, and trees with maximal nodes.
9
6
9


r
e
v
i
s
i
o
n
:
2
0
1
1
-
0
5
-
2
8







m
o
d
i
f
i
e
d
:
2
0
1
1
-
0
5
-
2
9


BOREL CONJECTURE AND DUAL BOREL CONJECTURE 11
(g) Moreover: Let T M,

E M, and let

E

be a lter system extending



E such that for all s
0
and 2
all A P() M we have: A (E
s
0
)
+
i A (E

s
0
)
+
. (In particular, this will be true for any

E

3
extending

E, provided that each E
s
0
is an M-ultralter.) 4
Then for each M we have T

E,s
0
= T

,s
0
(and hence T

E,s
0
M). (Proved by induction on .)
5
Now let A = (p
i
: i I) M be a maximal antichain in L
D
M , and assume (in V) that q L
D
. Let 6
s
0
stem(q).
7
We will show that q is compatible with some p
i
. This is clear if there is some i with s
0
p
i
and 8
stem(p
i
) s
0
, by (2.3). (In this case, p
i
q is a condition in L
D
with stem s
0
.)
9
So for the rest of the proof we assume that this is not the case, i.e.: 10
(2.5) There is no i with s
0
p
i
and stem(p
i
) s
0
.
Let J i I : s
0
stem(p
i
). We claim that there is j J with stem(p
j
) q (which as above implies 11
that q and p
j
are compatible).
12
Assume towards a contradiction that this is not the case. Then q is contained in the following tree T:
T (
<
)
[s
0
]
\
_
jJ
[stem(p
j
)] (2.6)
Note that T M. In V we have: 13
(2.7) The tree T contains a condition q with stem s
0
.
so by (e) (applied in V), followed by (g), and again by (e) (now in M) we get: 14
(2.8) The tree T also contains a condition p M with stem s
0
.
Now p has to be compatible with some p
i
. The sequences s
0
= stem(p) and stem(p
i
) have to be comparable, 15
so by (2.3) there are two possibilities:
16
(1) stem(p
i
) stem(p) = s
0
p
i
. We have excluded this case in our assumption (2.5).
17
(2) s
0
= stem(p) stem(p
i
) p. So i J. By construction of T (see (2.6)), we conclude stem(p
i
) T, 18
contradicting stem(p
i
) p T (see 2.8).
19
2.C. Ultralaver kills strong measure zero. The proof of the following lemma is a well-known routine 20
construction that works with many tree forcings. We will delay the proof until Lemma 2.34.
21
Lemma 2.9. If A is a nite set,

an L
D
-name, p L
D
, and p

A, then there is A and a pure 22


extension q
0
p such that q

= .
23
Denition 2.10. Let

be an increasing sequence of natural numbers. We say that X 2

is smz with 24
respect to

, if there exists a sequence (I
k
)
k
of basic intervals of 2

of measure 2

k
(i.e., each I
k
is of 25
the form [s
k
] for some s
k
2

k
) such that X
_
m
_
km
I
k
.
26
Remark 2.11. It is well known and easy to see that the properties
27
For all

there exists exists a sequence (I
k
)
k
of basic intervals of 2

of measure 2

k
such that 28
X
_
k
I
k
.
29
For all

there exists exists a sequence (I
k
)
k
of basic intervals of 2

of measure 2

k
such that 30
X
_
m
_
km
I
k
.
31
are equivalent. Hence, a set X is smz i X is smz with respect to all

.
32
The following lemma is a variant of the corresponding lemma (and proof) for Laver forcing (see for 33
example [Jec03, Lemma 28.20]): Ultralaver makes old uncountable sets non-smz.
34
Lemma 2.12. Let

D be a system of ultralters, and let

be the L
D
-name for the ultralaver real. Then each 35
uncountable set X V is forced to be non-smz (witnessed by the ultralaver real

).
36
More precisely, the following holds:
37
(2.13)
L
D
X V [2

1
(x
k
)
k
2

X
_
m
_
km
[x
k

k
].
We rst give two technical lemmas:
1
9
6
9


r
e
v
i
s
i
o
n
:
2
0
1
1
-
0
5
-
2
8







m
o
d
i
f
i
e
d
:
2
0
1
1
-
0
5
-
2
9


12 MARTIN GOLDSTERN, JAKOB KELLNER, SAHARON SHELAH, AND WOLFGANG WOHOFSKY
Lemma 2.14. Let p L
D
with stem s
<
, and let

x be a L
D
-name for a real in 2

. Then there exists a 2


pure extension q
0
p and a real 2

such that for every n , 3


(2.15) i succ
q
(s) : q
[s

i]

xn = n D
s
.
Proof. For each i succ
p
(s), let q
i

0
p
[s

i]
be such that q
i
decides

xi, i.e., there is a t


i
of length i such 4
that q
i

xi = t
i
(this is possible by Lemma 2.9).
5
Now we dene the real 2

as the D
s
-limit of the t
i
s. In more detail: For each n there is a 6
(unique)
n
2
n
such that i : t
i
n =
n
D
s
; since D
s
is a lter, there is a real 2

with n =
n
for 7
each n. Finally, let q
_
i
q
i
.
8
Lemma 2.16. Let p L
D
with stem s, and let (

x
k
)
k
be a sequence of L
D
-names for reals in 2

. Then 9
there exists a pure extension q
0
p and a family of reals (

)
q, s
2

such that for each q above s, 10


and every n , 11
(2.17) i succ
q
() : q
[

i]

n =

n D

.
Proof. We apply Lemma 2.14 to each node in p above s (and to

) separately: We rst get a p


1

0
p 12
and a
s
2

; for every immediate successor succ


p
1
(s), we get q


0
p
[]
1
and a

, and let 13
p
2

_

; in this way, we get a (fusion) sequence (p, p


1
, p
2
, . . .), and let q
_
k
p
k
.
14
Proof of Lemma 2.12. We want to prove (2.13). Assume towards a contradiction that X is an uncountable 15
set in V, and that (

x
k
)
k
is a sequence of names for reals in 2

and p L
D
such that 16
(2.18) p X
_
m
_
km
[

x
k

k
].
Let s
<
be the stem of p.
17
By Lemma 2.16, we can x a pure extension q
0
p and a family (

)
q, s
2

such that for each 18


q above the stem s and every n , condition (2.17) holds.
19
Since X is (in V and) uncountable, we can nd a real x

X which is dierent from each real in the 20


countable family (

)
q, s
; more specically, we can pick a family of natural numbers (n

)
q, s
such 21
that x

for any .
22
We can now nd r
0
q such that:
23
For all r above s and all i succ
r
() we have i > n

.
24
For all r above s and all i succ
r
() we have r
[

i]

.
25
So for all r above s we have, writing k for , that r
[

i]
forces x

x
k
n

] [

x
k

k
]. We conclude 26
that r forces x

_
ks
[

x
k

k
], contradicting (2.18).
27
Corollary 2.19. Let (t
k
)
k
be a dense subset of 2

.
28
Let

D be a system of ultralters, and let

be the L
D
-name for the ultralaver real. Then the set

H
_
m
_
km
[t
k

k
]
is forced to be a comeager set with the property that

H does not contain any translate of any old uncount- 29


able set. 30
Using Pawlikowskis Theorem 1.2 we get:
31
Corollary 2.20. There is a canonical name F for a closed null set such that X + F is positive for all 32
uncountable X in V. 33
In particular, no uncountable ground model set is smz in the ultralaver extension.
34
2.D. Thin sets and strong measure zero. For the notion of (very) thin set, we use an increasing func- 35
tion B

(k) (the function we use will be described in Corollary 3.3). We will assume that

= (

k
)
k
is 36
an increasing sequence of natural numbers with

k+1
B

(k). (We will later use a subsequence of the 37


ultralaver real

as

, see Lemma 2.22).


38
Denition 2.21. For X 2

and k we write X[

k
,

k+1
) for the set x[

k
,

k+1
) : x X. We say that
39
X 2

is very thin with respect to


and B

, if there are innitely many k with X[

k
,

k+1
) 40
B

(k).
1
9
6
9


r
e
v
i
s
i
o
n
:
2
0
1
1
-
0
5
-
2
8







m
o
d
i
f
i
e
d
:
2
0
1
1
-
0
5
-
2
9


BOREL CONJECTURE AND DUAL BOREL CONJECTURE 13
X 2

is thin with respect to


and B

, if X is the union of countably many very thin sets.


2
Note that the family of thin sets is a -ideal, while the family of very thin sets is not even an ideal. Also, 3
every very thin set is covered by a closed very thin (in particular nowhere dense) set. In particular, every 4
thin set is meager and the ideal of thin sets is a proper ideal.
5
Lemma 2.22. Let B

be an increasing function. Let



be an increasing sequence of natural numbers. We 6
dene a subsequence

of

in the following way:

k
=
n
k
where n
k+1
n
k
= B

(k) 2

k
. 7
Then we get: If X is thin with respect to

and B

, then X is smz with respect to



.
8
Proof. Assume that X =
_
i
Y
i
, each Y
i
very thin with respect to

and B

. Let (X
j
)
j
be an enumeration 9
of Y
i
: i where each Y
i
appears innitely often. So X
_
m
_
jm
X
j
.
10
By induction on j , we nd for all j > 0 some k
j
> k
j1
such that 11
X
j
[

k
j
,

k
j
+1
) B

(k
j
) hence X
j
[0,

k
j
+1
) B

(k
j
) 2

k
j
= n
k
j
+1
n
k
j
.
So we can enumerate X
j
[0,

k
j
+1
) as (s
i
)
n
k
j
i<n
k
j
+1
. Hence X
j
is a subset of
_
n
k
j
i<n
k
j
+1
[s
i
]; and each s
i
has 12
length

k
j
+1

i
, since

k
j
+1
=
n
k
j
+1
and i < n
k
j
+1
. This implies 13
X
_
m
_
jm
X
j

_
m
_
im
[s
i
]
Hence X is smz with respect to

.
14
Lemma 2.12 and Lemma 2.22 yield:
15
Corollary 2.23. Let B

be an increasing function. Let



D be a system of ultralters, and

the name for the 16


ultralaver real. Let

be constructed from B

and

as in Lemma 2.22. 17
Then L
D
forces that for every uncountable X 2

:
18
X is not smz with respect to

.
19
X is not thin with respect to

and B

.
20
2.E. Ultralaver and preservation of Lebesgue positivity. It is well known that both Laver forcing and 21
random forcing preserve Lebesgue positivity; in fact they satisfy a stronger property that is preserved 22
under countable support iterations. (So in particular, a countable support iteration of Laver and random 23
also preserves positivity.)
24
Ultralaver forcing L
D
will in general not preserve positivity. Indeed, if all ultralters D
s
are equal to the 25
same ultralter D

, then the range L


0
,
1
, . . . of the ultralaver real

will diagonalize D

, so every 26
ground model real x 2

(viewed as a subset of ) will either almost contain L or be almost disjoint to L, 27


which implies that the set 2

V of old reals is covered by a null set in the extension. However, later in 28


this paper it will become clear that if we choose the ultralters D
s
in a suciently generic way, then many 29
old positive sets will stay positive. More specically, in this section we will show (Lemma 2.29): If

D
M
is 30
we will show how a in a countable model M and r a random real over M, then we can nd an extension

D 31
such that L
D
forces that r remains random over M[H
M
] (and additionally some side conditions are met, 32
which are necessary to preserve the property in forcing iterations).
33
In Section 4.D we will see how to use this property to preserver randoms in limits.
34
The setup we use for preservation of randomness is basically the notation of Case A preservation 35
introduced in [She98, Ch.XVIII], see also [Gol93, GK06] or the textbook [BJ95, 6.1.B]:
36
Denition 2.24. We write for the collection of clopen sets on 2

. We say that the function Z : 37


is a code for a null set, if the measure of Z(n) is at most 2
n
for each n .
38
For such a code Z, the set nullset(Z) coded by Z is 39
nullset(Z)
_
n
_
kn
Z(k).
The set nullset(Z) obviously is a null set, and it is well known that every null set is contained in such a 40
set nullset(Z).
1
9
6
9


r
e
v
i
s
i
o
n
:
2
0
1
1
-
0
5
-
2
8







m
o
d
i
f
i
e
d
:
2
0
1
1
-
0
5
-
2
9


14 MARTIN GOLDSTERN, JAKOB KELLNER, SAHARON SHELAH, AND WOLFGANG WOHOFSKY
Denition 2.25. For a real r and any code Z, we dene Z
n
r by: 2
(k n) r Z(k).
We write Z r if Z
n
r holds for some n; i.e., if r nullset(Z).
3
For later reference, we record the following trivial fact: 4
(2.26) p

Z r i there is a name

n for an element of such that p

n
r.
Let P be a forcing notion, and

Z a P-name of a code for a null set. An interpretation of

Z below p is 5
some code Z

such that there is a sequence p = p


0
p
1
p
2
. . . such that p
m
forces

Zm = Z

m. 6
Usually we demand (which allows a simpler proof of the preservation theorem at limit stages) that the 7
sequence (p
0
, p
1
, . . . ) is inconsistent, i.e., p forces that there is an m such that p
m
G. Note that whenever 8
P adds a new -sequence of ordinals, we can nd such an interpretation for any

Z.
9
We now turn to preservation of Lebesgue positivity:
10
Denition 2.27. (1) Aforcing notion P preserves Borel outer measure, if P forces Leb

(A
V
) = Leb(A
V[G
P
]
) 11
for every code for a Borel set A. (Leb

denotes the outer Lebesgue measure, and for a Borel code 12


A and a set-theoretic universe V, A
V
denotes the Borel set coded by A in V.)
13
(2) P strongly preserves randoms, if the following holds: Let N H(

) be countable for a suciently 14


large regular cardinal

, let P, p,

Z N, let p P and let r be random over N. Assume that in N, 15


Z

is an interpretation of

Z, and assume Z

0
r. Then there is an N-generic q p forcing that r is 16
still random over N[G] and moreover,

Z
0
r. (In particular, P has to be proper.)
17
(3) Assume that P is absolutely denable. P strongly preserves randoms over countable models if (2) 18
holds for all countable (transitive
18
) models N of ZFC*.
19
It is easy to see that these properties are increasing in strength. (Of course (3)(2) works only if ZFC* 20
is satised in H(

).)
21
In [KS05] it is shown that (1) implies (3), provided that P is nep (non-elementary proper, i.e., nicely 22
denable and proper with respect to countable models). In particular, every Suslin ccc forcing notion such 23
as random forcing, and also many tree forcing notions including Laver forcing, are nep. However L
D
is not 24
nicely denable in this sense, as its denition uses ultralters as parameters.
25
Lemma 2.28. Both Laver forcing and random forcing strongly preserve randoms over countable models.
26
Proof. For random forcing, this is easy and well known (see, e.g., [BJ95, 6.3.12]).
27
For Laver forcing: By the above, it is enough to show (1). This was done by Woodin (unpublished) and 28
Judah-Shelah [JS90]. A nicer proof (including a variant of (2)) is given by Pawlikowski [Paw96b].
29
Ultralaver will generally not preserve Lebesgue positivity, let alone randomness. However, we get 30
the following local variant of strong preservation of randoms (which will be used in the preservation 31
theorem 4.33). The rest of this section will be devoted to the proof of the following lemma.
32
Lemma 2.29. Assume that M is a countable model,

D
M
an ultralter system in M and r a random real 33
over M. Then there is (in V) an ultralter system

D extending
19

D
M
, such that the following holds: 34
If
35
p L
D
M ,
36
in M,

Z = (

Z
1
, . . . ,

Z
m
) is a sequence of L
D
M-names for codes for null sets,
20
and Z

1
, . . . , Z

m
are 37
interpretations under p, witnessed by a sequence (p
n
)
n
with strictly increasing stems,
38
r is random over M,
39
Z

i

k
i
r for i = 1, . . . , m,
40
then there is a q p in L
D
forcing that
41
r is random over M[G
M
],
42

Z
i

k
i
r for i = 1, . . . , m.
1
18
later we will introduce ord-transitive models, and it is easy to see that it does not make any dierence whether we demand
transitive or not; this can be seen using a transitive collapse
19
This implies, by Lemma 2.4, that the L
D
-generic lter G induces an L
D
M-generic lter over M, which we call G
M
.
20
Recall that nullset(

Z) =
_
n
_
kn

Z(k) is a null set in the extension.


9
6
9


r
e
v
i
s
i
o
n
:
2
0
1
1
-
0
5
-
2
8







m
o
d
i
f
i
e
d
:
2
0
1
1
-
0
5
-
2
9


BOREL CONJECTURE AND DUAL BOREL CONJECTURE 15
For the proof of this lemma, we will use the following concepts:
2
Denition 2.30. Let p
<
be a tree. A front name below p is a function
21
h : F , where F p 3
is a front (a set that meets every branch of p in a unique point). (For notational simplicity we also allow h 4
to be dened on elements p; this way, every front name below p is also a front name below q whenever 5
q p.)
6
If h is a front name and

D is any lter system, we dene the corresponding L
D
-name (in the sense of
forcing)

z
h
by

z
h
( y, p
[s]
) : s F, y h(s). (2.31)
(This does not depend on the

D we use, since we set y ( x,
<
) : x y.)
7
Up to forced equality, the name

z
h
is characterized by the fact that p
[s]
forces (in any L
D
) that

z
h
= h(s), 8
for every s in the domain of h.
9
Note that the same object h can be viewed as a front name below p with respect to dierent forcings 10
L
D
1
, L
D
2
, as long as p L
D
1
L
D
2
.
11
Denition 2.32. Let p
<
be a tree. A continuous name below p is either of the following:
12
An -sequence of front names below p.
13
A -increasing function g : p
<
such that lim
n
lh(g(cn)) = for every branch c [p].
14
For each n, the set of minimal elements in s p : lh(g(s)) > n is a front, so each continuous name in the 15
second sense naturally denes a name in the rst sense, and conversely. Being a continuous name below p 16
does not involve the notion of nor does it depend on the lter system

D.
17
If g is a continuous name and

D is any lter system, we can again dene the corresponding L
D
-name

Z
g
(in the sense of forcing); we leave a formal denition of

Z
g
to the reader and content ourselves with this
characterization:
(s p) : p
[s]

L
D
g(s)

Z
g
. (2.33)
Note that a continuous name below p naturally corresponds to a continuous function F : [p]

, and 18

Z
g
is forced (by p) to be the value of F at the generic real

.
19
Lemma 2.34. L
D
has the following pure decision properties:
20
(1) Whenever

y is a name for an element of , p L


D
, then there is a pure extension p
1

0
p such 21
that

y =

z
h
(is forced) for a front name h below p
1
.
22
(2) Whenever

Y is a name for a sequence of elements of , p L


D
, then there is a pure extension 23
q
0
p such that

Y =

Z
g
(is forced) for some continuous name g below q.
24
(3) (This is Lemma 2.9.) If A is a nite set,

a name, p L
D
, and p forces

A, then there is A 25
and a pure extension q
0
p such that q

= .
26
Proof. Let p L
D
, s
0
stem(p),

y a name for an element of .


27
We call t p a good node in p if

y is a front name below p


[t]
(more formally: forced to be equal to

z
h
28
for a front name h). We can nd p
1

0
p such that for all t p
1
above s
0
: If there is q
0
p
[t]
1
such that t is 29
good in q, then t is already good in p
1
.
30
We claim that s
0
is now good (in p
1
). Note that for any bad node s the set t succ
p
1
(s) : t bad is 31
in D
+
s
. Hence, if s
0
is bad, we can inductively construct p
2

0
p
1
such that all nodes of p
2
are bad nodes 32
in p
1
. Now let q p
2
decide

y, s stem(q). Then q
0
p
[s]
1
, so s is good in p
1
, contradiction. This nishes 33
the proof of (1).
34
To prove (2), we rst construct p
1
as in (1) with respect to

y
0
. This gives a front F
1
p
1
deciding

y
0
. 35
Above each node in F
1
we nowrepeat the construction from(1) with respect to

y
1
, yielding p
2
, etc. Finally, 36
q
_
n
p
n
.
37
To prove (3): Similar to (1), we can nd p
1

0
p such that for each t p
1
: If there is a pure extension 38
of p
[t]
1
deciding

, then p
[t]
1
decides

; in this case we again call t good. Since there are only nitely many 39
possibilities for the value of

, any bad node t has D


+
t
many bad successors. So if the stem of p
1
is bad, we 40
can again reach a contradiction as in (1).
1
21
Instead of , the set of clopen sets, we may also consider other ranges of front names, such as the class of all ordinals, or the
set .
9
6
9


r
e
v
i
s
i
o
n
:
2
0
1
1
-
0
5
-
2
8







m
o
d
i
f
i
e
d
:
2
0
1
1
-
0
5
-
2
9


16 MARTIN GOLDSTERN, JAKOB KELLNER, SAHARON SHELAH, AND WOLFGANG WOHOFSKY
Corollary 2.35. Let

D be a lter system, and let G L
D
be generic. Then every Y

in V[G] is the 2
evaluation of a continuous name

Z
g
by G.
3
Proof. Assume that p L
D
forces that

Y is not the evaluation of a continuous name. We can nd q


0
p 4
and a continuous name g below q such that q

Y =

Z
g
. This is a contradiction.
5
We will need the following modication of the concept of continuous names.
6
Denition 2.36. Let p
<
be a tree, b [p] a branch. An almost continuous name below p (with 7
respect to b) is a -increasing function g : p
<
such that lim
n
lh(g(cn)) = for every branch 8
c [p], except possibly for c = b.
9
Note that except possibly for c = b is the only dierence between this denition and the denition of 10
a continuous name. 11
Since for any

D it is forced
22
that the generic real (for L
D
) is not equal to the exceptional branch b, we 12
again get a name

Z
g
of a function in

satisfying: 13
(s p) : p
[s]

L
D
g(s)

Z
g
.
An almost continuous name naturally corresponds to a continuous function F from [p] \ b into

.
14
Note that being an almost continuous name is a very simple combinatorial property of g which does 15
not depend on

D, nor does it involve the notion . Thus, the same function g can be viewed as an almost 16
continuous name for two dierent forcing notions L
D
1
, L
D
2
simultaneously.
17
Lemma 2.37. Let

D be a system of lters (not necessarily ultralters).
18
Assume that p = (p
n
)
n
witnesses that Y

is an interpretation of

Y, and that the lengths of the stems of 19


the p
n
are strictly increasing.
23
Then there exists a sequence q = (q
n
)
n
such that
20
(1) q
0
q
1
.
21
(2) q
n
p
n
for all n.
22
(3) q also interprets

Y as Y

. (This follows from the previous two statements.)


23
(4)

Y is almost continuous below q


0
, i.e., there is an almost continuous name g such that q
0
forces 24

Y =

Z
g
.)
25
(5)

Y is almost continuous below q


n
, for all n. (This follows from the previous statement.)
26
Proof. Let b be the branch described by the stems of the conditions p
n
: 27
b s : (n) s stem(p
n
).
We now construct a condition q
0
. For every s b satisfying stem(p
n
) s stem(p
n+1
) we set 28
succ
q
0
(s) = succ
p
n
(s), and for all t succ
q
0
(s) except for the one in b we let q
[t]
0

0
p
[t]
n
be such that

Y is 29
continuous below q
[t]
0
. We can do this by Lemma 2.34(2).
30
Now we set 31
q
n
p
n
q
0
= q
[stem(p
n
)]
0
p
n
.
This takes care of (1) and (2). Now we show (4): Any branch c of q
0
not equal to b must contain a node 32
s

k b with s b, so c is a branch in q
[s

k]
0
, below which

Y was continuous.
33
The following lemmas and corollaries are the motivation for considering continuous and almost contin- 34
uous names. 35
Lemma 2.38. Let

D be a system of lters (not necessarily ultralters). Let p L
D
, let b be a branch, 36
and let g : p
<
be an almost continuous name below p with respect to b; write

Z
g
for the associated 37
L
D
-name.
38
Let r 2

be a real, n
0
. Then the following are equivalent:
39
(1) p
L
D
r
_
nn
0

Z
g
(n), i.e.,

Z
g

n
0
r.
40
(2) For all n n
0
and for all s p for which g(s) has length > n we have r g(s)(n).
41
Note that (2) does not mention the notion and does not depend on

D.
1
22
This follows from our assumption that all our lters contain the Fr echet lter.
23
It is easy to see that for every L
D
-name

Y we can nd such p and Y

: First nd p which interprets both

Y and

, and then thin


out to get a strictly increasing sequence of stems.
9
6
9


r
e
v
i
s
i
o
n
:
2
0
1
1
-
0
5
-
2
8







m
o
d
i
f
i
e
d
:
2
0
1
1
-
0
5
-
2
9


BOREL CONJECTURE AND DUAL BOREL CONJECTURE 17
Proof. (2) (1): Assume that there is s p for which g(s) = (C
0
, . . . , C
n
, . . . , C
k
) and r C
n
. Then 2
p
[s]
forces that the generic sequence

Z
g
= (

Z(0),

Z(1), . . .) starts with C


0
, . . . , C
n
, so p
[s]
forces r

Z
g
(n).
3
(1) (2): Assume that p does not force r
_
nn
0

Z
g
(n). So there is a condition q p and some 4
n n
0
such that q r

Z
g
(n). By increasing the stem of q, if necessary, we may assume that s stem(q) 5
is not on b (the exceptional branch), and that g(s) has already length > n. Let C
n
g(s)(n) be the n-th 6
entry of g(s). So p
[s]
already forces

Z
g
(n) = C
n
; now q
[s]
p
[s]
, and q
[s]
forces the following statements: 7
r

Z
g
(n),

Z
g
(n) = C
n
. Hence r C
n
, so (2) fails.
8
Corollary 2.39. Let

D
1
and

D
2
be systems of lters, and assume that p is in L
D
1
L
D
2
. Let g : p
<
9
be an almost continuous name of a sequence of clopen sets, and let

Z
g
1
and

Z
g
2
be the associated L
D
1
-name 10
and L
D
2
-name, respectively.
11
Then for any real r and n we have 12
p
L
D
1

Z
g
1

n
r p
L
D
2

Z
g
2

n
r.
(We will use this corollary for the special case that L
D
1
is an ultralaver forcing, and L
D
2
is Laver forcing.)
13
Lemma 2.40. Let

D
1
and

D
2
be systems of lters, and assume that p is in L
D
1
L
D
2
. Let g : p
<
be 14
a continuous name of a sequence of clopen sets, let F p be a front and let h : F be a front name. 15
Again we will write

Z
g
1
,

Z
g
2
for the associated names of codes for null sets, and we will write

n
1
and

n
2
for 16
the associated L
D
1
- and L
D
2
-names, respectively, of natural numbers.
17
Then for any real r we have: 18
p
L
D
1

Z
g
1

n
1
r p
L
D
2

Z
g
2

n
2
r.
Proof. Assume p
L
D
1

Z
g
1

n
1
r. So for each s F we have: p
[s]

L
D
1

Z
g
1

h(s)
r. By Corollary 2.39, we 19
also have p
[s]

L
D
2

Z
g
2

h(s)
r. So also p
[s]

L
D
2

Z
g
2

n
2
r for each s F. Hence p
L
D
2

Z
g
2

n
2
r.
20
Corollary 2.41. Assume q L forces in Laver forcing that

Z
g
k
r for k = 1, 2, . . ., where each g
k
is a 21
continuous name of a code for a null set. Then there is a Laver condition q

0
q such that for all ultralter 22
systems

D we have:
23
If q

L
D
, then q

forces (in ultralaver forcing L


D
) that

Z
g
k
r for all k.
24
Proof. By (2.26) we can nd a sequence (

n
k
)

k=1
of L-names such that q

Z
g
k

n
k
r for each k. By 25
Lemma 2.34(2) we can nd q

0
q be such that this sequence is continuous below q

. Since each

n
k
is 26
now a front name below q

, we can apply the previous lemma.


27
Lemma 2.42. Let M be a countable model, r 2

,

D
M
M an ultralter system,

D a lter system 28
extending

D
M
, q L
D
. For any V-generic lter G L
D
we write G
M
for the (M-generic, by Lemma 2.4) 29
lter on L
D
M .
30
The following are equivalent:
31
(1) q
L
D
r is random over M[G
M
].
32
(2) For all names

Z M of codes for null sets: q


L
D

Z r.
33
(3) For all continuous names g M: q
L
D

Z
g
r.
34
Proof. (1)(2) holds because every null set is contained in a set of the form nullset(Z), for some code Z.
35
(2)(3): Every code for a null set in M[G
M
] is equal to

Z
g
[G
M
], for some g M, by Corollary 2.35.
36
Lemma 2.43. Let r be random over a countable model M. Then there is a countable model M

M such 37
that (2

)
M
is countable in M

, but r is still random over M

.
38
Proof. We will need the following forcing notions, all dened in M: 39
M
C
//
B
1

M
C

B
2

M
B
1

P=C

B
2
/B
1
//
M
C

B
2
Let C be the forcing that collapses the continuum to with nite conditions.
1
9
6
9


r
e
v
i
s
i
o
n
:
2
0
1
1
-
0
5
-
2
8







m
o
d
i
f
i
e
d
:
2
0
1
1
-
0
5
-
2
9


18 MARTIN GOLDSTERN, JAKOB KELLNER, SAHARON SHELAH, AND WOLFGANG WOHOFSKY
Let B
1
be random forcing (trees T 2
<
of positive measure).
2
Let

B
2
be the C-name of random forcing.
3
Let i : B
1
C

B
2
be the natural complete embedding T (1
C
, T).
4
Let

P be a B
1
-name for the forcing C

B
2
/i[G
B
1
], the quotient of C

B
2
by the complete subforcing 5
i[B
1
].
6
The random real r is B
1
-generic over M. In M[r] we let P

P[r]. Now let H P be generic over M[r]. 7


Then r H B
1

P C

B
2
induces an M-generic lter J C and an M[J]-generic lter K

B
2
[J]; it 8
is easy to check that K interprets the

B
2
-name of the canonical random real as the given random real r.
9
Hence r is random over the countable model M

M[J], and (2

)
M
is countable in M

. 10
M
J
//
r

M[J]
K

M[r]
H
//
M[r][H]
11
Proof of Lemma 2.29. We will rst describe a construction that deals with a single triple ( p,

Z,

Z

) (where 12
p is a sequence of conditions with strictly increasing stems which interprets

Z as

Z

); this construction will 13


yield a condition q

= q

( p,

Z,

Z

). We will then show how to deal with all possible triples.


14
So let p = p
0
be a condition, and let p = (p
k
)
k
be a sequence interpreting

Z as

Z

, where the lengths 15


of the stems of p
n
are strictly increasing. It is easy to see that it is enough to deal with a single null set, i.e., 16
m = 1, and with k
1
= 0. We write

Z and Z

instead of

Z
1
and Z

.
17
Using Lemma 2.37 we may (strengthening the conditions in our interpretation) assume (in M) that the 18
sequence (

Z(k))
k
is almost continuous, witnessed by g : p
<
. By Lemma 2.43, we can nd a model 19
M

M such that (2

)
M
is countable in M

, but r is still random over M

.
20
We now work in M

. Note that g still denes an almost continuous name, which we again call

Z.
21
Each lter in D
M
s
is now countably generated; let A
s
be a pseudo-intersection of D
M
s
which additionally 22
satises A
s
succ
p
(s) for all s p above the stem. Let D

s
be the Fr echet lter on A
s
. Let p

L
D
be the 23
tree with the same stem as p which satises succ
p
(s) = A
s
for all s p

above the stem.


24
By Lemma 2.4, we know that L
D
M is an M-complete subforcing of L
D
(in M

as well as in V.) We write 25


G
M
for the induced lter on L
D
M .
26
We now work in V. Note that below the condition p

, the forcing L
D
is just Laver forcing L, and that
p

L
p. Using Lemma 2.28 we can nd a condition q p

(in Laver forcing L) such that:


q is M

-generic. (2.44)
q
L
r is random over M

[G
L
] (hence also over M[G
M
]). (2.45)
Moreover, q
L

Z
0
r. (2.46)
Enumerate all continuous L
D
M -names for codes for null sets from M as

Z
g
1
,

Z
g
2
, . . . Applying Corol- 27
lary 2.41 yields a condition q

q such that for all lter systems



E satisfying q

L
E
, we have q

L
E
28

Z
g
i
r for all i. Corollary 2.39 and Lemma 2.42 now implies: 29
(2.47)
For every lter system

E satisfying q

L
E
, q

forces in L
E
that r is random over M
and that

Z
0
r.
By thinning out q

we may assume that 30


(2.48) For each

M there is k such that k q

.
We have now described a construction of q

= q

( p,

Z, Z

).
31
Let ( p
n
,

Z
n
, Z
n
) enumerate all triples ( p,

Z, Z

) M where p interprets

Z as Z

(and consists of con- 32


ditions with strictly increasing stems). For each n write
n
for
_
k
stem(p
n
k
), the branch determined by the 33
stems of the sequence p
n
. We now dene by induction a sequence q
n
of conditions:
34
q
0
= q

( p
0
,

Z
0
, Z
0
).
35
Given q
n1
and ( p
n
,

Z
n
, Z
n
), we nd k
0
such that
n
k
0
q
0
q
n1
(using (2.48)). Let k
1
be 36
such that stem(p
n
k
1
) has length > k
0
. We replace p
n
by p

(p
n
k
)
kk
1
. (Obviously, p

still interprets 37

Z
n
as Z
n
.) Now let q
n
q

( p

Z
n
, Z
n
).
1
9
6
9


r
e
v
i
s
i
o
n
:
2
0
1
1
-
0
5
-
2
8







m
o
d
i
f
i
e
d
:
2
0
1
1
-
0
5
-
2
9


BOREL CONJECTURE AND DUAL BOREL CONJECTURE 19
Note that the stem of q
n
is at least as long as the stem of p
n
k
1
, and is therefore not in q
0
q
n1
, so 2
stem(q
i
) and stem(q
j
) are incompatible for all i j. Therefore we can choose for each s an ultralter D
s
3
extending D
M
s
such that stem(q
i
) s implies succ
q
i (s) D
s
.
4
Note that all q
i
are in L
D
. Therefore, we can use (2.47). Also, q
i
p
i
0
.
5
3. Jxxts roacrxo 6
In this section, we dene a family of forcing notions that has two faces (hence the name Janus forcing): 7
Elements of this family may be countable (and therefore equivalent to Cohen), and they may also be 8
essentially random.
9
We will need the following properties of the Janus forcing notions J:
10
Wish List 3.1. Throughout the whole paper we x a function B

: given by Corollary 3.3. The 11


Janus forcings will depend on a real parameter

= (

m
)
m

which grows fast with respect to B

. (In 12
our application,

will be given by a subsequence of an ultralaver real.)


13
The sequence

and the function B

together dene a notion of a thin set, see Denition 2.21.


14
(1) There is a canonical J-name for a (code for a) null set

. Whenever X 2

is not thin, and J is 15


countable, then J forces that X is not strongly meager, witnessed by nullset(

). Moreover, for any 16


-centered J-name of a forcing

Q, also J

Q forces that X is not strongly meager, again witnessed 17


by nullset(

). (See Denitions 2.21 and 2.24, as well as Lemma 3.9.)


18
(2) Canonical above in particular means: If M is a countable model of ZFC*, J
M
M is a Janus 19
forcing which M-completely embeds into the Janus forcing J, then in the J-extension V[H],

[H] 20
is the same as

Z
M

[H
M
], where H
M
is the induced lter on J
M
.
21
(3) Let M be a countable transitive model and J
M
a Janus forcing in M. Then J
M
is a Janus forcing in 22
V as well (and of course countable in V). (See Fact 3.8.)
23
(4) Whenever M is a countable transitive model, J
M
is a Janus forcing in M, then there is a Janus 24
forcing J such that
25
J is (in V) equivalent to random forcing (actually we just need some forcing that strongly 26
preserves random reals)
27
J
M
is an M-complete subforcing of J.
28
(See Lemma 3.16.)
29
(Advice to the reader: At rst reading, you may want to take these properties as granted, and skip to 30
Section 4.)
31
3.A. Denition of Janus. A Janus forcing J will consist of:
32
A countable core (or: backbone) which is dened in a combinatorial way from a parameter

. 33
(In our application, we will use a Janus forcing immediately after an ultralaver forcing, and

will 34
be a subsequence of the ultralaver real.) This core is of course equivalent to Cohen forcing.
35
Some additional stung J \ (countable
24
or uncountable). We allow great freedomfor this, we 36
just require that the core is a suciently complete subforcing (in a specic combinatorial sense).
37
We will use the following combinatorial theorem from [BS10]:
38
Lemma 3.2 ([BS10, Theorem 8]
25
). For every , > 0 there exists N
,
such that for all suciently 39
large nite sets I there is a nonempty family 7
I
consisting of sets A 2
I
,
A
2
I
such that if X 2
I
, 40
X N
,
then 41
A 7
I
: X + A = 2
I

7
I

1 .
(Recall that X + A x + a : x X, a A.)
42
Rephrasing and specializing to =
1
4
and =
1
2
i
we get:
1
24
Also the trivial case J = is allowed.
25
The theorem in [BS10] actually says for a suciently large I, but the proof shows that this should be read as for all suciently
large I.
9
6
9


r
e
v
i
s
i
o
n
:
2
0
1
1
-
0
5
-
2
8







m
o
d
i
f
i
e
d
:
2
0
1
1
-
0
5
-
2
9


20 MARTIN GOLDSTERN, JAKOB KELLNER, SAHARON SHELAH, AND WOLFGANG WOHOFSKY
Corollary 3.3. For every i there exists B

(i) such that for all nite sets I with I B

(i) there is a 2
nonempty family 7
I
satisfying the following:
3
7
I
consists of sets A 2
I
with
A
2
I

1
2
i
.
4
For every X 2
I
satisfying X B

(i), the set A 7


I
: X + A = 2
I
has at least
3
4
7
I
elements.
5
Assumption 3.4. We x a suciently fast increasing sequence

= (

i
)
i
of natural numbers; more 6
precisely, the sequence

will be a subsequence of an ultralaver real



, dened as in Lemma 2.22 using the 7
function B

from Corollary 3.3. Note that in this case

i+1

i
B

(i); so we can x for each i a family 8


7
i
P(2
L
i
) on the interval L
i
[

i
,

i+1
) according to Corollary 3.3.
9
Denition 3.5. First we dene the core =

of our forcing: 10
=
_
i
_
j<i
7
j
.
In other words, i = (A
0
, . . . , A
i1
) for some i , A
0
7
0
, . . . , A
i1
7
i1
. We will denote the 11
number i by height().
12
The forcing notion is ordered by reverse inclusion (i.e., end extension): if .
13
Denition 3.6. Let

= (

i
)
i
be as in the assumption above. We say that J is a Janus forcing based on

14
if: 15
(1) (, ) is an incompatibility-preserving subforcing of J.
16
(2) For each i the set : height() = i is predense in J. So in particular, J adds a 17
branch through . The union of this branch is called

= (

0
,

1
,

2
, . . .), where

i
2
L
i
with 18

i
7
i
.
19
(3) Fatness:
26
For all p J and all real numbers > 0 there are arbitrarily large i such that there 20
is a core condition = (A
0
, . . . , A
i1
) (of length i) with
21
A 7
i
:

A j
J
p
7
i

1 .
(4) J is ccc.
22
(5) J is separative.
23
(6) (To simplify some technicalities:) J H(
1
).
24
We now dene

, which will be a canonical J-name for (a code for) a null set. We will use the sequence 25

added by J (see Denition 3.6(2)).


26
Denition 3.7. Each

i
denes a clopen set

i
= x 2

: xL
i

i
of measure at most
1
2
i+1
. The 27
sequence

= (

0
,

1
,

2
, . . .) is (a name for) a code for the null set 28
nullset(

) =
_
n<
_
in

Z

i
.
For simplicity, we will write

instead of nullset(

).
29
For later reference, we record the following trivial fact:
30
Fact 3.8. Let (M
n
)
n
be an increasing sequence of countable models, and let J
n
M
n
be Janus forcings. 31
Assume that J
n
is M
n
-complete in J
n+1
. Then
_
n
J
n
is a Janus forcing, and an M
n
-complete extension of 32
J
n
for all n. 33
This is also true if all M
n
are equal. In other words: If J M is a Janus forcing in M, then J is also a 34
Janus forcing in V.
1
26
Actually, (3) implies (2).
9
6
9


r
e
v
i
s
i
o
n
:
2
0
1
1
-
0
5
-
2
8







m
o
d
i
f
i
e
d
:
2
0
1
1
-
0
5
-
2
9


BOREL CONJECTURE AND DUAL BOREL CONJECTURE 21
3.B. Janus and strongly meager. Carlson [Car93] showed that Cohen reals make every uncountable 2
set X of the ground model not strongly meager in the extension (and that not being strongly meager is 3
preserved in a subsequent forcing with precaliber
1
). We show that a countable Janus forcing J does the 4
same (for a subsequent forcing that is even -centered, not just precaliber
1
). This sounds trivial, since 5
any (nontrivial) countable forcing is equivalent to Cohen forcing anyway. However, we show (and will 6
later use) that the canonical null set

dened above witnesses that X is not strongly meager (and not 7


just some null set that we get out of the isomorphism between J and Cohen forcing). The point is that 8
while is not a complete subforcing of J, the condition (3) of the Denition 3.6 guarantees that Carlsons 9
argument still works, if we assume that X is non-thin (not just uncountable). This is enough for us, since 10
by Corollary 2.23 ultralaver forcing makes any uncountable set non-thin.
11
Recall that we xed the increasing sequence

= (

i
)
i
and B

. In the following, whenever we say 12


(very) thin we mean (very) thin with respect to

and B

(see Denition 2.21).


13
Lemma 3.9. If X is not thin, J is a countable Janus forcing based on

, and

R is a J-name for a -centered 14


forcing notion, then J

R forces that X is not strongly meager witnessed by the null set

.
15
Proof. Let

c be a J-name for a function

c :

R witnessing that

R is -centered.
16
Recall that

witnesses that X is not strongly meager means that X +

= 2

. Assume towards 17
a contradiction that (p, r) J

R forces that X +

. Then we can x a (J

R)-name

such that 18
(p, r)

X +

, i.e., (p, r) (x X)

x +

. By denition of

, we get 19
(p, r) (x X) (n ) (i n)

L
i
xL
i
+

i
.
For each x X we can nd (p
x
, r
x
) (p, r) and natural numbers n
x
and m
x
such that p
x
forces 20
that

c(r
x
) = m
x
and 21
(p
x
, r
x
) (i n
x
)

L
i
xL
i
+

i
.
So X =
_
pJ,m,n
X
p,m,n
, where X
p,m,n
is the set of all x with p
x
= p, m
x
= m, n
x
= n. (Note that J is 22
countable, so the union is countable.) As X is not thin, there is some p

, m

, n

such that X

X
p

,m

,n
is 23
not very thin. So we get for all x X

: 24
(3.10) (p

, r
x
) (i n

L
i
xL
i
+

i
.
Since X

is not very thin, there is some i


0
such that for all i i
0
25
(3.11) the (nite) set X

L
i
has more than B

(i) elements.
Due to the fact that J is a Janus forcing (see Denition 3.6 (3)), there are arbitrarily large i such that 26
there is a core condition = (A
0
, . . . , A
i1
) with 27
(3.12)
A 7
i
:

A j
J
p

7
i


2
3
.
Fix such an i larger than both i
0
and n

, and x a condition satisfying (3.12).


28
We now consider the following two subsets of 7
i
: 29
(3.13) A 7
i
:

A j
J
p

and A 7
i
: X

L
i
+ A = 2
L
i
.
By (3.12), the relative measure (in 7
i
) of the left one is at least
2
3
; due to (3.11) and the denition of 7
i
30
according to Corollary 3.3, the relative measure of the right one is at least
3
4
; so the two sets in (3.13) are 31
not disjoint, and we can pick an A belonging to both.
32
Clearly,

A forces (in J) that

i
is equal to A. Fix q J witnessing

A j
J
p

. Then 33
(3.14) q
J
X

L
i
+

i
= X

L
i
+ A = 2
L
i
.
Since p

forces that for each x X

the color

c(r
x
) = m

, we can nd an r

which is (forced by q p

34
to be) a lower bound of the nite set r
x
: x X

, where X

is any nite set with X

L
i
= X

L
i
.
35
By (3.10), 36
(q, r

L
i
X

L
i
+

i
= X

L
i
+

i
,
contradicting (3.14).
1
9
6
9


r
e
v
i
s
i
o
n
:
2
0
1
1
-
0
5
-
2
8







m
o
d
i
f
i
e
d
:
2
0
1
1
-
0
5
-
2
9


22 MARTIN GOLDSTERN, JAKOB KELLNER, SAHARON SHELAH, AND WOLFGANG WOHOFSKY
Corollary 3.15. Let X be uncountable. If L
D
is any ultralaver forcing adding an ultralaver real

, and

2
is dened from

as in Lemma 2.22, and if

J is a countable Janus forcing based on


Q is any -centered 3
forcing, then L
D

Q forces that X is not strongly meager.


4
3.C. Janus may be random. We show here that every countable Janus forcing can be embedded into a 5
Janus forcing which is equivalent to random forcing.
6
Lemma 3.16. Let M be a countable model of ZFC*, and let J
M
M be a Janus forcing (based on

). 7
Then there is a Janus forcing J (based on

) such that
8
J
M

M
J (i.e., J
M
is an M-complete subforcing of J).
9
J is forcing equivalent to random forcing.
10
Proof. Let r
n
: n be an enumeration of J
M
J
M
. Let D
k
: k be an enumeration of all D M 11
which are open dense subsets of J
M
. Recall that =
J
M
was dened in Denition 3.5.
12
We now x n for a while (up to (3.18)). We will construct a nitely splitting tree S
n

<
and a family 13
(
n
s
, p
n
s
,
n
s
)
sS
n satisfying the following (suppressing the superscript n):
14
(a)
s
,
()
= (), s t implies
s

t
, and s
S
n t implies
s


t
. 15
(So in particular the set
t
: t succ
S
n (s) is a (nite) antichain above
s
in .)
16
(b) p
s
J
M
, p
()
= r
n
; if s t then p
t

J
M p
s
(hence p
t
r
n
); s
S
n t implies p
s

J
M p
t
.
17
(c) p
s

J
M
s
.
18
(d)
s

s
, and
t
: t succ
S
n (s) is the set of all succ

s
) which are compatible with p
s
.
19
(e) The set
t
: t succ
S
n (s) is a subset of succ

s
) of relative size at least 1
1
lh(s)+10
.
20
(f) Each s S
n
has at least 2 successors (in S
n
).
21
(g) If k = lh(s), then p
s
D
k
(and therefore also in all D
l
for l < k).
22
Set
()
= () and p
()
= r
n
. Given s,
s
and p
s
, we construct succ
S
n (s) and (
t
, p
t
)
tsucc
S
n (s)
: We apply 23
fatness 3.6(3) to p
s
with =
1
lh(s)+10
. So we get some

s
of height bigger than the height of
s
such 24
that the set B of elements of succ

s
) which are compatible with p
s
has relative size at least 1 . Since 25
p
s

J
M
s
we get that

s
is compatible with (and therefore stronger than)
s
. Enumerate B as
0
, . . . ,
l1
. 26
Set succ
S
n (s) = s

i : i < l and
s

i
=
i
. For t succ
S
(s), choose p
t
J
M
stronger than both
t
and p
s
27
(which is obviously possible since
t
and p
s
are compatible), and moreover p
t
D
lh(t)
. This concludes the 28
construction of the family (
n
s
, p
n
s
,
n
s
)
sS
n .
29
So (S
n
, ) is a nitely splitting nonempty tree of height with no maximal nodes and no isolated 30
branches. [S
n
] is the (compact) set of branches of S
n
. The closed subsets of [S
n
] are exactly the sets of 31
the form [T], where T S
n
is a subtree of S
n
with no maximal nodes. [S
n
] carries a natural (uniform) 32
probability measure
n
, which is characterized by 33

n
((S
n
)
[t]
) =
1
succ
S
n (s)

n
((S
n
)
[s]
)
for all s S
n
and all t succ
S
n (s). (We just write
n
(T) instead of
n
([T]) to increase readability.)
34
We call T S
n
positive if
n
(T) > 0, and we call T pruned if (T
[s]
) > 0 for all s T. (Clearly every 35
tree T contains a pruned tree T

of the same measure, which can be obtained from T by removing all nodes 36
s with (T
[s]
) = 0.)
37
Let T S
n
be a positive, pruned tree and > 0. Then on all but nitely many levels k there is an s T 38
such that 39
(3.17) succ
T
(s) succ
S
n (s) has relative size 1 .
(This follows from Lebesgues density theorem, or can easily be seen directly: Set C
m
=
_
tT, lh(t)=m
(S
n
)
[t]
. 40
Then C
m
is a decreasing sequence of closed sets, each containing [T]. If the claim fails, then
n
(C
m+1
)) 41

n
(C
m
) (1 ) innitely often; so
n
(T) (
_
m
C
m
) = 0.)
42
It is well known that the set of positive, pruned subtrees of S
n
, ordered by inclusion, is forcing equivalent 43
to random forcing (which can be dened as the set of positive, pruned subtrees of 2
<
).
44
We have now constructed S
n
for all n. Dene
J = J
M

_
n
_
(n, T) : T S
n
is a positive pruned tree
_
(3.18)
with the following partial order:
1
9
6
9


r
e
v
i
s
i
o
n
:
2
0
1
1
-
0
5
-
2
8







m
o
d
i
f
i
e
d
:
2
0
1
1
-
0
5
-
2
9


BOREL CONJECTURE AND DUAL BOREL CONJECTURE 23
The order on J extends the order on J
M
. 2
(n

, T

) (n, T) if n = n

and T

T.
3
(n, T) p if there is a k such that p
n
t
p for all t T of length k. (Note that this will then be true 4
for all bigger k as well.)
5
p (n, T) never holds.
6
The lemma now easily follows from the following properties:
7
(1) The order on J is transitive.
8
(2) J
M
is an incompatibility-preserving subforcing of J. 9
In particular, J satises item (1) of Denition 3.6 of Janus forcing.
10
(3) For all k: the set (n, T
[t]
) : t T, lh(t) = k is a (nite) predense antichain below (n, T).
11
(4) (n, T
[t]
) is stronger than p
n
t
for each t T (witnessed, e.g., by k = lh(t)). Of course, (n, T
[t]
) is 12
stronger than (n, T) as well.
13
(5) Since p
n
t
D
k
for k = lh(t), this implies that each D
k
is predense below each (n, S
n
) and therefore 14
in J. 15
So in particular, J
M

M
J, i.e., J
M
is an M-complete subforcing of J. 16
Also, for each k the set : height() = k is in M (and predense in J
M
). Therefore this set is 17
predense in J as well, i.e., item (2) of the Denition 3.6 of Janus forcing is satised.
18
(6) The condition (n, S
n
) is stronger than r
n
, so (n, S
n
) : n is predense in J and J \ J
M
is dense 19
in J. 20
Below each (n, S
n
), the forcing J is isomorphic to random forcing. 21
Therefore, J itself is forcing equivalent to random forcing. (In fact, the complete Boolean algebra 22
generated by J is isomorphic to the standard random algebra, Borel sets modulo null sets.)
23
(7) The remaining item of the denition of Janus forcing, fatness 3.6(3), is satised. 24
I.e., given (n, T) J and > 0 there is an arbitrarily high

such that the relative size of the 25


set succ

) : j (n, T) is at least 1 . (We will show (1 )


2
instead, to simplify the 26
notation.)
27
We show (7): Given (n, T) J and > 0, we use (3.17) to get an arbitrarily high s T such that succ
T
(s) 28
is of relative size 1 in succ
S
(s). We may choose s of length >
1

. We claim that

s
is as required:
29
Let B
t
: t succ
S
(s). Note that B = succ

s
) : j p
s
.
30
B has relative size 1
1
lh(s)
1 in succ

s
) (according to property (e) of S
n
).
31
C
t
: t succ
T
(s) is a subset of B of relative size 1 according to our choice of s.
32
So C is of relative size (1 )
2
in succ

s
).
33
Each
t
C is compatible with (n, T), as (n, T
[t]
) p
t

t
(see (4)).
34
It is easy (but not even necessary) to check that J is separative. In any case, we could replace
J
by

J
, 35
thus making J separative without changing
J
M, since J
M
was already separative.
36
4. Aimosr rrxrrr xxo ximosr cotxrxair strroar rrraxrroxs 37
A main tool to construct the forcing for BC+dBC will be partial countable support iterations, more 38
particularly almost nite support and almost countable support iterations. A partial countable support 39
iteration is a forcing iteration (P

, Q

)
<
2
such that for each limit ordinal the forcing notion P

is a subset 40
of the countable support limit of (P

, Q

)
<
which satises some natural properties (see Denition 4.6).
41
Instead of transitive models, we will use ord-transitive models (which are transitive when ordinals are 42
considered as urelements). Why do we do that? We want to approximate the generic iteration

P of length 43

2
with countable models; this can be done more naturally with ord-transitive models (since obviously 44
countable transitive models only see countable ordinals). We call such an ord-transitive model a candi- 45
date (provided it satises some nice properties, see Denition 4.2). A basic point is that forcing extensions 46
work naturally with candidates.
47
In the following, x = (M
x
,

P
x
) will denote a pair such that M
x
is a candidate and

P
x
is (in M
x
) a partial 48
countable support iteration; similarly we write, e.g., y = (M
y
,

P
y
) or x
n
= (M
x
n
,

P
x
n
).
49
We will need the following results to prove BC+dBC. (However, other than in the case of the ultralaver 50
and Janus section, the reader will probably have to read this section to understand the construction in the 51
next section, and not just the wish list.)
1
9
6
9


r
e
v
i
s
i
o
n
:
2
0
1
1
-
0
5
-
2
8







m
o
d
i
f
i
e
d
:
2
0
1
1
-
0
5
-
2
9


24 MARTIN GOLDSTERN, JAKOB KELLNER, SAHARON SHELAH, AND WOLFGANG WOHOFSKY
Wish List 4.1. Given x = (M
x
,

P
x
), we can construct by induction on a partial countable support iteration 2

P = (P

, Q

)
<
2
satisfying:
3
There is a canonical M
x
-complete embedding from

P
x
to

P.
4
In this construction, we can use at each stage any desired Q

, as long as P

forces that Q
x

is (evaluated 5
as) an M
x
[H
x

]-complete subforcing of Q

(where H
x

P
x

is the M
x
-generic lter induced by the generic 6
lter H

). 7
Moreover, we can demand either of the following two additional properties of this limit:
27
8
(1) If all Q

are forced to be -centered, and Q

is trivial for all M


x
, then P

2
is -centered.
9
(2) If r is random over M
x
, and all Q

locally preserve randomness of r over M


x
[H
x

] (see Deni- 10
tion 4.31), then also P

2
locally preserves the randomness of r.
11
Actually, we need the following variant: Assume that we already have P

0
for some
0
M
x
, and that P
x

0
12
canonically embeds into P

0
, and that the respective assumption on Q

holds for all


0
. Then we get 13
that P

0
forces that the quotient P

2
/P

0
satises the respective conclusion.
14
We also need:
28
15
(3) If instead of a single x we have a sequence x
n
such that each P
x
n
canonically (and M
x
n
-completely) 16
embeds into P
x
n+1
, then we can nd a partial countable support iteration

P into which all P
x
n
embed 17
canonically (and we can again use any desired Q

, assuming that Q
x
n

is an M
x
n
[H
x
n

]-complete 18
subforcing of Q

for all n ).
19
(4) (A fact that is easy to prove but awkward to formulate.) If a -system argument produces two 20
x
1
, x
2
as in Lemma 5.7(3), then we can nd a partial countable support iteration

P such that

P
x
i
21
canonically (and M
x
i
-completely) embeds into

P for i = 1, 2.
22
4.A. Ord-transitive models. We will use ord-transitive models, as introduced in [She04] (see also the 23
presentation in [Kel]). We briey summarize the basic denitions and properties (restricted to the rather 24
simple case needed in this paper):
25
Denition 4.2. Fix a suitable nite subset ZFC

of ZFC (that is satised by H(

) for suciently large 26


regular

).
27
(1) A set M is called candidate, if
28
M is countable,
29
(M, ) is a model of ZFC

,
30
M is ord-absolute: M = Ord i Ord, for all M,
31
M is ord-transitive: if x M \ Ord, then x M,
32
+ 1 M. 33
is a limit ordinal and = + 1 are both absolute between M and V.
34
(2) A candidate M is called nice, if has countable conality and the countable set A is conal 35
in both are absolute between M and V. (So if M has countable conality, then M is 36
conal in .) Moreover, we assume
1
M (which implies
1
M
=
1
) and
2
M (but we do 37
not require
2
M
=
2
).
38
(3) Let P
M
be a forcing notion in a candidate M. (To simplify notation, we can assume without loss 39
of generality that P
M
Ord = (or at least ) and that therefore P
M
M and also A M 40
whenever M thinks that A is a subset of P
M
.) Recall that a subset H
M
of P
M
is M-generic (or: 41
P
M
-generic over M), if A H
M
= 1 for all maximal antichains A in M.
42
(4) Let H
M
be P
M
-generic over M and

a P
M
-name in M. We dene the evaluation

[H
M
]
M
to be x if 43
M thinks that p
P
M

= x for some p H
M
and x M (or equivalently just for x MOrd), and 44

[H
M
]
M
: (

, p)

, p H
M
otherwise. Abusing notation we write

[H
M
] instead of

[H
M
]
M
, 45
and we write M[H
M
] for

[H
M
] :

is a P
M
-name in M.
46
(5) The ord-collapse k (or k
M
) is a recursively dened function with domain M: k(x) = x if x Ord, 47
and k(x) = k(y) : y x M otherwise.
1
27
The -centered version is central for the proof of dBC; the random preserving version for BC.
28
This will give -closure and
2
-cc for the preparatory forcing R.
9
6
9


r
e
v
i
s
i
o
n
:
2
0
1
1
-
0
5
-
2
8







m
o
d
i
f
i
e
d
:
2
0
1
1
-
0
5
-
2
9


BOREL CONJECTURE AND DUAL BOREL CONJECTURE 25
(6) The ord-transitive closure of a set x is dened inductively on the rank: 2
ordclos(x) = x
_
ordclos(y) : y x \ Ord.
So ordclos(x) is the smallest ord-transitive set containing x as a subset. HCON is the collection of 3
all sets x such that the ord-transitive closure of x is countable. x is in HCON i x is element of 4
some candidate. In particular, all reals and all ordinals are HCON.
5
We write HCON

for the family of all sets x in HCON whose ord-transitive closure (or, in this 6
case equivalently, transitive closure) only contains ordinals < .
7
Fact 4.3. (1) The ord-collapse of a countable elementary submodel of H(

) is a nice candidate.
8
(2) Unions, intersections etc. are generally not absolute for candidates. For example, let x M \ Ord. 9
In M we can construct a set y such that M = y =
1
x. Then y is not an ordinal and therefore a 10
subset of M, and in particular y is countable and y
1
x.
11
(3) Let j : M M

be the transitive collapse of a candidate M, and f :


1
M

Ord the inverse 12


(restricted to the ordinals). Obviously M

is a countable transitive model of ZFC

; moreover M 13
is characterized by the pair (M

, f ) (we call such a pair a labeled transitive model). Note that f 14


satises f ( + 1) = f () + 1, f () = for . M = ( is a limit) i f () is a limit. 15
M = cf() = i cf( f ()) = , and in that case f [] is conal in . On the other hand, given 16
a transitive countable model M

of ZFC

and an f as above, then we can construct a (unique) 17


candidate M corresponding to (M

, f ).
18
(4) All candidates M with M Ord
1
are hereditarily countable, so their number is at most 2

0
. 19
Similarly, the cardinality of HCON

is at most continuum whenever <


2
.
20
(5) If H
M
is P
M
-generic over M, then M[H
M
] is a candidate as well and an end-extension of M such 21
that M Ord = M[H
M
] Ord. If M is nice and (M thinks that) P
M
is proper, then M[H
M
] is nice 22
as well. 23
(6) Forcing extensions commute with the transitive collapse j: If M corresponds to (M

, f ), then 24
H
M
P
M
is P
M
-generic over M i H

j[H
M
] is P

j(P
M
)-generic over M

, and in that case 25


M[H
M
] corresponds to (M

[H

], f ). In particular, the forcing extension of M[H


M
] of M satises 26
the forcing theorem (everything that is forced is true, and everything true is forced).
27
(7) For elementary submodels, forcing extensions commute with ord-collapses: Let N be a countable 28
elementary submodel of H(

), P N, k : N M the ord-collapse (so M is a candidate), and let 29


H be P-generic over V. Then H is P-generic over N i H
M
k[H] is P
M
k(P)-generic over M; 30
and in that case the ord-collapse of N[H] is M[H
M
].
31
Assume that a nice candidate M thinks that (

P
M
,

Q
M
) is a forcing iteration of length
2
V
(we will 32
usually write
2
for the length of the iteration, by this we will always mean
2
V
and not the possibly 33
dierent
2
M
). In this section, we will construct an iteration (

P,

Q) in V, also of length
2
, such that each 34
P
M

canonically and M-completely embeds into P

for all
2
M. Once we know (by induction) that 35
P
M

M-completely embeds into P

, we know that a P

-generic lter H

induces a P
M

-generic (over M) 36
lter which we call H
M

. Then M[H
M

] is a candidate, but nice only if P


M

is proper. We will not need that 37


M[H
M

] is nice, actually we will only investigate set of reals (or elements of H(


1
)) in M[H
M

], so it does 38
not make any dierence whether we use M[H
M

] or its transitive collapse.


39
Remark 4.4. In the discussion so far we omitted some details regarding the theory ZFC

(that a candidate 40
has to satisfy). The following ne print hopefully absolves us from any liability. (It is entirely irrelevant 41
for the understanding of the paper.)
42
We have to guarantee that each M[H
M

] that we consider satises enough of ZFC to make our arguments 43


work (for example, the denitions and basic properties of ultralaver and Janus forcings should work). This 44
turns out to be easy, since (as usual) we do not need the full power set axiom for these arguments (just the 45
existence of, say,
5
). So it is enough that each M[H
M

] satises some xed nite subset of ZFC minus 46


power set, which we call ZFC

.
47
Of course we can also nd a bigger (still nite) set ZFC

that implies:
10
exists, and each forcing 48
extension of the universe with a forcing of size
4
satises ZFC

. And it is provable (in ZFC) that each 49


H() satises ZFC

for suciently large regular .


50
We dene candidate using the weaker theory ZFC

, and require that nice candidates satises the stronger 51


theory ZFC

. This guarantees that all forcing extensions (by small forcings) of nice candidates will be 1
9
6
9


r
e
v
i
s
i
o
n
:
2
0
1
1
-
0
5
-
2
8







m
o
d
i
f
i
e
d
:
2
0
1
1
-
0
5
-
2
9


26 MARTIN GOLDSTERN, JAKOB KELLNER, SAHARON SHELAH, AND WOLFGANG WOHOFSKY
candidates (in particular, satisfy enough of ZFC such that our arguments about Janus or ultralaver forcings 2
work). Also, every ord-collapse of a countable elementary submodel N of H() will be a nice candidate.
3
4.B. Partial countable support iterations. We introduce the notion of partial countable support limit: 4
a subset of the countable support (CS) limit containing the union (i.e., the direct limit) and satisfying some 5
natural requirements.
6
Let us rst describe what we mean by forcing iteration. They have to satisfy the following require- 7
ments: 8
A topless forcing iteration (P

, Q

)
<
is a sequence of forcing notions P

and P

-names Q

9
of quasiorders with a weakest element 1
Q

. A topped iteration additionally has a nal limit P

. 10
Each P

is a set of partial functions on (as, e.g., in [Gol93]). More specically, if < 11


and p P

, then p P

. Also, p
P

p() Q

for all dom(p). The order on P

will 12
always be the natural one: q p i q forces (in P

) that q
tot
() p
tot
() for all < , where 13
r
tot
() = r() for all dom(r) and 1
Q

otherwise. P
+1
consists of all p with p P

and 14
p p
tot
() Q

, so it is forcing equivalent to P

.
15
P

whenever < . (In particular, the empty condition is an element of each P

.)
16
For any p P

and any q P

( < ) with q p, the partial function q p q p[, ) is a 17


condition in P

as well (so in particular, p is a reduction of p, hence P

is a complete subforcing 18
of P

; and q p is the weakest condition in P

stronger than both q and p).


19
Abusing notation, we usually just write

P for an iteration (be it topless or topped).
20
We usually write H

for the generic lter on P

(which induces P

-generic lters called H

for 21
). For topped iterations we call the lter on the nal limit sometimes just H instead of H

.
22
We use the following notation for factorization of iterations:
23
For < , in the P

-extension V[H

], we let P

/H

be the set of all p P

with p H

24
(ordered as in P

). We may occasionally write P

/P

for the P

-name of P

/H

.
25
Since P

is a complete subforcing of P

, this is a factorization with the usual properties, in partic- 26


ular P

is equivalent to P

(P

/H

).
27
Denition 4.5. Let

P be a (topless) iteration of limit length . We dene three limits of

P:
28
The direct limit is the union of the P

(for < ). So this is the smallest possible limit of the 29


iteration. 30
The inverse limit consists of all partial functions p with domain such that p P

for all 31
< . This is the largest possible limit of the iteration.
32
The full countable support limit P
CS

of

P is the inverse limit if cf() = and the direct limit 33
otherwise. 34
We say that P

is a partial CS limit, if P

is a subset of the full CS limit and the sequence (P

is a 35
topped iteration. In particular, this means that P

contains the direct limit, and satises the following for 36


each < : P

is closed under p p, and whenever p P

, q P

, q p, then also the partial 37


function q p is in P

.
38
So for a given topless

P there is a well-dened inverse, direct and full CS limit. If cf() > , then they 39
all coincide. If cf() = , then the direct limit and the full CS limit (=inverse limit) dier. Both of them 40
are partial CS limits, but there are many more possibilities for partial CS limits. By denition, all of them 41
will yield iterations.
42
Note that the name CS limit is slightly inappropriate, as the size of supports of condition is not part of 43
the denition. To give a more specic example: Consider a topped iteration

P of length + where P

is 44
the direct limit and P
+
is the full CS limit. Let p be any element of the full CS limit of

P which is not 45
in P

; then p is not in P
+
either. So not every countable subset of + can appear as the support of a 46
condition. 47
Denition 4.6. A forcing iteration

P is called a partial CS iteration, if
48
every limit is a partial CS limit, and
49
every Q

is (forced to be) separative


29
, i.e.,
Q

coincides with

.
1
29
The reason for this requirement is briey discussed in Section 7
9
6
9


r
e
v
i
s
i
o
n
:
2
0
1
1
-
0
5
-
2
8







m
o
d
i
f
i
e
d
:
2
0
1
1
-
0
5
-
2
9


BOREL CONJECTURE AND DUAL BOREL CONJECTURE 27
The following fact can easily be proved by transnite induction:
2
Fact 4.7. Let

P be a partial CS iteration. Then for all the forcing notion P

is separative.
3
From now on, all iterations we consider will be partial CS iterations. In this paper, we will only be 4
interested in proper partial CS iterations, but properness is not part of the denition of partial CS iteration. 5
(The reader may safely assume that all iterations are proper.)
6
Note that separativity of the Q

implies that all partial CS iterations satisfy the following (trivially 7


equivalent) properties:
8
Fact 4.8. Let

P be a topped partial CS iteration of length . Then:
9
(1) Let H be P

-generic. Then p H i p H

for all < .


10
(2) For all q, p P

: If q

p for each < , then q

p.
11
(3) For all q, p P

: If q

p for each < , then q j p.


12
We will be concerned with the following situation:
13
Assume that M is a nice candidate,

P
M
is (in M) a topped partial CS iteration of length (a limit ordinal 14
in M), and

P is (in V) a topless partial CS iteration of length

sup( M). (Recall that cf() = 15


is absolute between M and V, and that cf() = implies

= .) Moreover, assume that we already have 16


a system of M-complete coherent
30
embeddings i

: P
M

for

M = M. (Recall that any 17


potential partial CS limit of

P is a subforcing of the full CS limit P
CS

.) It is easy to see that there is only one 18


possibility for an embedding j : P
M

P
CS

(in fact, into any potential partial CS limit of



P) that extends 19
the i

s naturally:
20
Denition 4.9. For a topped partial CS iteration

P
M
in M of length and a topless one

P in V of length 21

sup(M) together with coherent embeddings i

, we dene j : P
M

P
CS

, the canonical extension, 22


in the obvious way: Given p P
M

, take the sequence of restrictions to M-ordinals, apply the functions i

, 23
and build j(p) from the resulting coherent sequence.
24
We do not claim that j : P
M

P
CS

is M-complete.
31
In the following, we will construct partial CS 25
limits P

such that j : P
M

is M-complete. (Obviously, one requirement for such a limit is that 26


j[P
M

] P

.) We will actually dene two versions: The almost FS and the almost CS limit.
27
Instead of arbitrary systems of embeddings i

, we will only be interested in canonical ones. We as- 28


sume for notational convenience that Q
M

is a subset of Q

(this will naturally be the case in our application 29


anyway).
30
Denition 4.10 (The canonical embedding). Let

P be a partial CS iteration in V and

P
M
a partial CS 31
iteration in M, both topped and of length M. We construct by induction on ( + 1) M the 32
canonical M-complete embeddings i

: P
M

. More precisely: We try to construct them, but it 33


is possible that the construction fails. If the construction succeeds, then we say that

P
M
(canonically) 34
embeds into

P, or the canonical embeddings work, or just:

P is over

P
M
, or over P
M

.
35
Let = + 1. By induction hypothesis, i

is M-complete, so a V-generic lter H

induces 36
an M-generic lter H
M

i
1

[H

] P
M

. We require that (in the H

extension) the set Q


M

[H
M

] is 37
an M[H
M

]-complete subforcing of Q

[H

]. In this case, we dene i

in the obvious way.


38
For limit, let i

be the canonical extension of the family (i

)
M
. (If

sup( M) < , 39
then i

is a map into P

, which is a complete subforcing of P

.) We require that P

contains the 40
range of i

, and that i

is M-complete; otherwise the construction fails.


41
In this section we try to construct a partial CS iteration

P (over a given

P
M
) satisfying additional prop- 42
erties. 1
30
I.e., they commute with the restriction maps: i

(p) = i

(p) for < and p P


M

.
31
For example, if =

= and if P
M

is the nite support limit of a nontrivial iteration, then j : P


M

P
CS

is not complete: In
M, let c
n
be (a P
M
n
-name for) a nontrivial element of Q
M
n
(i.e., 1
Q
M
n

c
n
). Let p
n
be the P
M
n
-condition c
m
H(m) for all m < n
and let q
n
be the P
M
n+1
-condition (c
0
, . . . , c
n1
, c
n
) = p
n
p
n+1
, i.e., n is minimal with c
n
H(n). In M, the set A = q
n
: n
is a maximal antichain in P
M

. Moreover, the sequence (p


n
)
n
is a decreasing coherent sequence, therefore i
n
(p
n
) denes an element
p

in P
CS

, which is clearly incompatible with all j(q


n
), hence j[A] is not maximal.
9
6
9


r
e
v
i
s
i
o
n
:
2
0
1
1
-
0
5
-
2
8







m
o
d
i
f
i
e
d
:
2
0
1
1
-
0
5
-
2
9


28 MARTIN GOLDSTERN, JAKOB KELLNER, SAHARON SHELAH, AND WOLFGANG WOHOFSKY
Remark 4.11. What is the role of

sup( M)? When our inductive construction of



P arrives at 2
P

where

< , it would be too late


32
to take care of M-completeness of i

at this stage, even if all 3


i

work nicely for M. Note that

< implies that is uncountable in M, and that therefore 4


P
M

=
_
M
P
M

. So the natural extension j of the embeddings (i

)
M
has range in P

, which will be 5
a complete subforcing of P

. So we have to ensure M-completeness already in the construction of P

.
6
For now we just record:
7
Lemma 4.12. Assume that we have topped iterations

P
M
(in M) of length and

P (in V) of length

8
sup( M), and that for all M the canonical embedding i

: P
M

works. Let i

: P
M

P
CS

9
be the canonical extension. 10
(1) If P
M

is (in M) a direct limit (which is always the case if has uncountable conality) then i

11
(might not work, but at least) has range in P

and preserves incompatibility.


12
(2) If i

has a range contained in P

and maps predense sets D P


M

in M to predense sets i

[D] 13
P

, then i

preserves incompatibility (and therefore works).


14
Proof. (1) Since P
M

is a direct limit, the canonical extension i

has range in
_
<
P

, which is subset of 15
any partial CS limit P

. Incompatibility in P
M

is the same as incompatibility in P


M

for suciently large 16


M, so it by assumption it is preserved by i

and hence also by i

.
17
(2) Fix p
1
, p
2
P
M

, and assume that their images are compatible in P

; we have to show that they are 18


compatible in P
M

. So x a generic lter H P

containing i

(p
1
) and i

(p
2
).
19
In M, we dene the following set D: 20
D q P
M

: (q p
1
q p
2
) or ( < : q
P
M

p
1
) or ( < : q
P
M

p
2
).
Using Fact 4.8(3) it is easy to check that D is dense. Since i

preserves predensity, there is q D 21


such that i

(q) H. We claim that q is stronger than p


1
and p
2
. Otherwise we would have without loss 22
of generality q
P
M

p
1
for some < . But the lter H contains both i

(q) and i

(p
1
), 23
contradicting the assumption that i

preserves incompatibility.
24
4.C. Almost nite support iterations. Recall Denition 4.9 (of the canonical extension) and the setup 25
that was described there: We have to nd a subset P

of P
CS

such that the canonical extension j : P


M

26
is M-complete.
27
We now dene the almost nite support limit. (The direct limit will in general not do, as it may not 28
contain the range j[P
M

]. The almost nite support limit is the obvious modication of the direct limit, and 29
it is the smallest partial CS limit P

such that j[P


M

] P

, and it indeed turns out to be M-complete as 30


well.)
31
Denition 4.13. Let be a limit ordinal in M, and let

sup( M). Let



P
M
be a topped iteration in M 32
of length , and let

P be a topless iteration in V of length

. Assume that the canonical embeddings i

33
work for all M =

M. Let i

be the canonical extension. We dene the almost nite support 34


limit of

P over

P
M
(or: almost FS limit) as the following subforcing P

of P
CS

: 35
P

q i

(p) P
CS

: p P
M

and ( M) q
P

(p) .
Note that for cf() > , the almost FS limit is equal to the direct limit, as each p P
M

is in fact in P
M

36
for some M, so i

(p) = i

(p) P

.
37
Lemma 4.14. Assume that

P and

P
M
are as above and let P

be the almost FS limit. Then



P

is a 38
partial CS iteration, and i

works, i.e., i

is an M-complete embedding from P


M

to P

. (As P

is a complete 39
subforcing of P

, this also implies that i

is M-complete from P
M

to P

.)
1
32
For example: Let =
1
and

=
1
M. Assume that P
M

1
is (in M) a (or: the unique) partial CS limit of a nontrivial
iteration. Assume that we have a topless iteration

P of length

in V such that the canonical embeddings work for all


1
M.
If we set P

to be the full CS limit, then we cannot further extend it to any iteration of length
1
such that the canonical embedding
i

1
works: Let p

and q

be as in footnote 31. In M, the set A = q

:
1
is a maximal antichain, and the sequence (p

1
is a decreasing coherent sequence. But in V there is an element p

P
CS

with p

= p

for all M. This condition p

is
clearly incompatible with all elements of j[A] = j(p

) : M. Hence j[A] is not maximal.


9
6
9


r
e
v
i
s
i
o
n
:
2
0
1
1
-
0
5
-
2
8







m
o
d
i
f
i
e
d
:
2
0
1
1
-
0
5
-
2
9


BOREL CONJECTURE AND DUAL BOREL CONJECTURE 29
Proof. It is easy to see that P

is a partial CS limit and contains the range i

[P
M

]. We now show preserva- 2


tion of predensity; this implies M-completeness by Lemma 4.12.
3
Let (p
j
)
jJ
M be a maximal antichain in P
M

, and let q i

(p) be a condition in P

. (If

< , i.e., if 4
cf() > , then we can choose p to be the empty condition.) Fix M be such that q P

. Let H

be 5
P

-generic and contain q, so p is in H


M

. Now in M[H
M

] the set p
j
: j J, p
j
P
M

/H
M

is predense 6
in P
M

/H
M

(since this is forced by the empty condition in P


M

). In particular, p is compatible with some p


j
, 7
witnessed by p

p, p
j
in P
M

/H
M

.
8
We can nd q

q deciding j and p

; since certainly q

(p

), we may assume even without 9


loss of generality. Now q

(p

) q i

(p) (since q

q and p

p), and q

(p

) i

(p
j
) (since 10
p

p
j
).
11
Denition and Claim 4.15. Let

P
M
be a topped partial CS iteration in M of length . We can construct by 12
induction on + 1 an almost nite support iteration

P over

P
M
(or: almost FS iteration) as follows:
13
(1) As induction hypothesis we assume that the canonical embedding i

works for all M. (So 14


the notation M[H
M

] makes sense.)
15
(2) Let = + 1. If M, then we can use any Q

provided that (it is forced that) Q


M

is an 16
M[H
M

]-complete subforcing of Q

. (If M, then there is no restriction on Q

.)
17
(3) Let M and cf() = . Then P

is the almost FS limit of (P

, Q

)
<
over P
M

.
18
(4) Let M and cf() > . Then P

is again the almost FS limit of (P

, Q

)
<
over P
M

(which 19
also happens to be the direct limit).
20
(5) For limit ordinals not in M, P

is the direct limit.


21
So the claim includes that the resulting

P is a (topped) partial CS iteration of length over

P
M
(i.e., 22
the canonical embeddings i

work for all ( + 1) M), where we only assume that the Q

satisfy the 23
obvious requirement. (Note that we can always nd some suitable Q

for M, for example we can just 24


take Q
M

itself.)
25
Proof. We have to show (by induction) that the resulting sequence

P is a partial CS iteration, and that

P
M
26
embeds into

P. For successor cases, there is nothing to do. So assume that is a limit. If P

is a direct 27
limit, it is trivially a partial CS limit; if P

is an almost FS limit, then the easy part of Lemma 4.14 shows 28


that it is a partial CS limit.
29
So it remains to show that for a limit M, the (naturally dened) embedding i

: P
M

is 30
M-complete. This was the main claim in Lemma 4.14.
31
The following lemma is natural and easy.
32
Lemma 4.16. Assume that we construct an almost FS iteration

P over

P
M
where each Q

is (forced to be) 33
ccc. Then P

is ccc (and in particular proper).


34
Proof. We show that P

is ccc by induction on . For successors, we use that Q

is ccc. For of 35
uncountable conality, we know that we took the direct limit coboundedly often (and all P

are ccc for 36


< ), so by a result of Solovay P

is again ccc. For a limit of countable conality not in M, just use 37


that all P

are ccc for < , and the fact that P

is the direct limit. This leaves the case that M has 38


countable conality, i.e., the P

is the almost FS limit. Let A P

be uncountable. Each a A has the 39


form q i

(p) for p P
M

and q
_
<
P

. We can thin out the set A such that p are the same and all q 40
are in the same P

. So there have to be compatible elements in A.


41
All almost FS iterations that we consider in this paper will satisfy the countable chain condition (and 42
hence in particular be proper).
43
We will need a variant of this lemma for -centered forcing notions.
44
Lemma 4.17. Assume that we construct an almost FS iteration

P over

P
M
where only countably many Q

45
are nontrivial (e.g., only those with M) and where each Q

is (forced to be) -centered. Then P

is 46
-centered as well. 47
Proof. By induction: The direct limit of countably many -centered forcings is -centered, as is the almost 48
FS limit of -centered forcings (to color q i

(p), use p itself together with the color of q).


1
9
6
9


r
e
v
i
s
i
o
n
:
2
0
1
1
-
0
5
-
2
8







m
o
d
i
f
i
e
d
:
2
0
1
1
-
0
5
-
2
9


30 MARTIN GOLDSTERN, JAKOB KELLNER, SAHARON SHELAH, AND WOLFGANG WOHOFSKY
We will actually need two variants of the almost FS construction: Countably many models M
n
; and 2
starting the almost FS iteration with some
0
.
3
Firstly, we can construct an almost FS iteration not just over one iteration

P
M
, but over an increasing 4
chain of iterations. Analogously to Denition 4.13 and Lemma 4.14, we can show:
5
Lemma 4.18. For each n , let M
n
be a nice candidate, and let

P
n
be a topped partial CS iteration 6
in M
n
of length
33
M
0
of countable conality, such that M
m
M
n
and M
n
thinks that

P
m
canonically 7
embeds into

P
n
, for all m < n. Let

P be a topless iteration of length into which all

P
n
canonically embed.
8
Then we can dene the almost FS limit P

over (

P
n
)
n
as follows: Conditions in P

are of the form 9


q i
n

(p) where n , p P
n

, and q P

for some M
n
. Then P

is a partial CS limit over each



P
n
.
10
As before, we get the following corollary:
11
Corollary 4.19. Given M
n
and

P
n
as above, we can construct a topped partial CS iteration

P such that 12
each

P
n
embeds M
n
-completely into it; we can choose Q

as we wish (subject to the obvious restriction 13


that each Q
n

is an M
n
[H
n

]-complete subforcing). If we always choose Q

to be ccc, then

P is ccc; this is 14
the case if we set Q

to be the union of the (countable) sets Q


n

.
15
Proof. We can dene P

by induction. If
_
n
M
n
has countable conality, then we use the almost 16
FS limit as in Lemma 4.18. Otherwise we use the direct limit. If M
n
has uncountable conality, 17
then

sup( M) is an element of M
n+1
. In our induction we have already considered

and have 18
dened P

by Lemma 4.18 (applied to the sequence (



P
n+1
,

P
n+2
, . . .)). This is sucient to show that 19
i
n

: P
n

is M
n
-complete.
20
Secondly, we can start the almost FS iteration after some
0
(i.e.,

P is already given up to
0
, and we 21
can continue it as an almost FS iteration up to ), and get the same properties that we previously showed 22
for the almost FS iteration, but this time for the quotient P

/P

0
. In more detail:
23
Lemma 4.20. Assume that

P
M
is in M a (topped) partial CS iteration of length , and that

P is in V a 24
topped partial CS iteration of length
0
over

P
M

0
for some
0
M. Then we can extend

P to a 25
(topped) partial CS iteration of length over

P
M
, as in the almost FS iteration (i.e., using the almost FS 26
limit at limit points >
0
with M of countable conality; and the direct limit everywhere else). We 27
can use any Q

for
0
(provided Q
M

is an M[H
M

]-complete subforcing of Q

). If all Q

are ccc, then 28


P

0
forces that P

/H

0
is ccc (in particular proper); if moreover all Q

are -centered and only countably 29


many are nontrivial, then P

0
forces that P

/H

0
is -centered.
30
4.D. Almost countable support iterations. Almost countable support iterations

P (over a given itera- 31
tion

P
M
in a candidate M) will have the following two crucial properties: There is a canonical M-complete 32
embedding of

P
M
into

P, and

P preserves a given random real (similar to the usual countable support 33
iterations).
34
Denition and Claim 4.21. Let

P
M
be a topped partial CS iteration in M of length . We can construct by 35
induction on + 1 the almost countable support iteration

P over

P
M
(or: almost CS iteration):
36
(1) As induction hypothesis, we assume that the canonical embedding i

works for every M. 37


We set
34
38
(4.22) min(M \ ),

sup( + 1 : M).
Note that

.
39
(2) Let = + 1. We can choose any desired forcing Q

; if M it is of course required that Q


M

is 40
an M[H
M

]-complete subforcing of Q

. This denes P

.
41
(3) Let cf() > . Then P

is the direct limit.


42
(4) Let cf() = and assume that M (so M is conal in and

= = ). We dene P

= P

43
as the union of the following two sets:
44
The almost FS limit of (P

, Q

)
<
, see Denition 4.13.
1
33
Or only: M
n
0
for some n
0
.
34
So for successors M, we have

= = . For M limit, = and

is as in Denition 4.9.
9
6
9


r
e
v
i
s
i
o
n
:
2
0
1
1
-
0
5
-
2
8







m
o
d
i
f
i
e
d
:
2
0
1
1
-
0
5
-
2
9


BOREL CONJECTURE AND DUAL BOREL CONJECTURE 31
The set P
gen

of M-generic conditions q P
CS

, i.e., those which satisfy 2


q
P
CS

i
1

[H
P
CS

] P
M

is M-generic.
(5) Let cf() = and assume that M but M is conal in , so

= < . We dene P

= P

3
as the union of the following two sets:
4
The direct limit of (P

, Q

)
<
.
5
The set P
gen

of M-generic conditions q P
CS

, i.e., those which satisfy 6


q
P
CS

i
1

[H
P
CS

] P
M

is M-generic.
(Note that the M-generic conditions form an open subset of P
CS

= P
CS

.)
7
(6) Let cf() = and M not conal in (so M). Then P

is the full CS limit of (P

, Q

)
<
8
(see Denition 4.5).
9
So the claim is that for every choice of Q

(with the one obvious restriction), this construction always 10


results in a partial CS iteration

P over

P
M
. The proof is a bit cumbersome; it is a variant of the usual proof 11
that properness is preserved in countable support iterations (see e.g. [Gol93]).
12
We will use the following fact in M (for the iteration

P
M
): 13
(4.23)
Let

P be a topped iteration of length . Let
1

2
. Let p
1
be a P

1
-name
for a condition in P

, and let D be an open dense set of P

. Then there is a P

2
-name
p
2
for a condition in D such that the empty condition of P

2
forces: p
2
p
1
and:
if p
1
is in P

/H

2
, then the condition p
2
is as well.
The following easy fact will also be useful: 14
(4.24)
Let P be a subforcing of Q. We dene Pp r P : r p. Assume that p P
and Pp = Qp.
Then for any P-name

x and any formula (x) we have: p


P
(

x) i p
Q
(

x).
We now prove by induction on the following statement (which includes that the Denition and 15
Claim 4.21 works up to ). Let ,

be as in (4.22).
16
Lemma 4.25. (a) The topped iteration

P of length is a partial CS iteration.
17
(b) The canonical embedding i

: P
M

works, hence also i

: P
M

works.
18
(c) Moreover, assume that
19
M ,
20

p M is a P
M

-name of a P
M

-condition,
21
q P

forces (in P

) that

p[H
M

] is in H
M

.
22
Then there is a q
+
P

(and therefore in P

) extending q and forcing that

p[H
M

] is in H
M

.
23
Proof. First let us deal with the trivial cases. It is clear that we always get a partial CS iteration.
24
Assume that =
0
+ 1 M, i.e., =

= . It is clear that i

works. To get q
+
, rst extend q to 25
some q

0
(by induction hypothesis), then dene q
+
extending q

by q
+
(
0
)

p(
0
).
26
If =
0
+ 1 M, there is nothing to do.
27
Assume that cf() > (whether M or not). Then

< . So i

: P
M

works by induction, 28
and similarly (c) follows from the inductive assumption. (Use the inductive assumption for =

; 29
the that we got at that stage is the same as the current , and the q
+
we obtained at that stage will 30
still satisfy all requirements at the current stage.)
31
Assume that cf() = and that M is bounded in . Then the proof is the same as in the 32
previous case.
33
We are left with the cases corresponding to (4) and (5) of Denition 4.21: cf() = and M is conal 34
in . So either M, then

= = , or M, then

= < and cf() > .


35
We leave it to the reader to check that P

is indeed a partial CS iteration. The main point is to see that 36


q p P

whenever q p. If p P
gen

, then this follows because P


gen

is open in P
CS

; the other cases 37


are immediate from the denition (by induction).
38
We now turn to claim (c). Assume q P

and

p M are given, M .
39
Let (D
n
)
n
enumerate all dense sets of P
M

which lie in M, and let (


n
)
n
be a sequence of ordinals in 1
M which is conal in , where
0
= .
9
6
9


r
e
v
i
s
i
o
n
:
2
0
1
1
-
0
5
-
2
8







m
o
d
i
f
i
e
d
:
2
0
1
1
-
0
5
-
2
9


32 MARTIN GOLDSTERN, JAKOB KELLNER, SAHARON SHELAH, AND WOLFGANG WOHOFSKY
2
Using (4.23) in M, we can nd a sequence (

p
n
)
n
satisfying the following in M, for all n > 0:
3

p
0
=

p.
4

p
n
M is a P
M

n
-name of a P
M

-condition in D
n
.
5

P
M
n

p
n

P
M

p
n1
.
6

P
M
n
If

p
n1

n
H
M

n
, then

p
n

n
H
M

n
as well.
7
Using the inductive assumption for the
n
s, we can now nd a sequence (q
n
)
n
of conditions satisfying 8
the following:
9
q
0
= q, q
n
P

n
.
10
q
n

n1
= q
n1
.
11
q
n

P
n

p
n1

n
H
M

n
, so also

p
n

n
H
M

n
.
12
Let q
+
P
CS

be the union of the q


n
. Then for all n:
13
(1) q
n

P
CS

p
n

n
H
M

n
, so also q
+
forces this. 14
(Using induction on n.)
15
(2) For all n and all m n: q
+

P
CS

p
m

m
H
M

m
, so also

p
n

m
H
M

m
. 16
(As

p
m

p
n
.)
17
(3) q
+

P
CS

p
n
H
M

. 18
(Recall that P
CS

is separative, see Fact 4.7. So i

p
n
) H

i i

n
(

p
m
) H

m
for all large m.)
19
As q
+

P
CS

p
n
D
n
H
M

, we conclude that q
+
P
gen

(using 4.12, applied to P


CS

). In particular, P
gen

20
is dense in P

: Let q i

(p) be an element of the almost FS limit; so q P

for some < . Now nd a 21


generic q
+
extending q and stronger than i

(p), then q
+
q i

(p).
22
It remains to show that i

is M-complete. Let A M be a maximal antichain of P


M

, and p P

. Assume 23
towards a contradiction that p forces in P

that i
1

[H

] does not intersect A in exactly one point.


24
Since P
gen

is dense in P

, we can nd some q p in P
gen

. Let 25
P

r P
CS

: r q = r P

: r q,
where the equality holds because P
gen

is open in P
CS

.
26
Let be the canonical name for a P

-generic lter, i.e.: ( r, r) : r P

. Let R be either P
CS

or P

. 27
We write ()
R
for the lter generated by in R, i.e., ()
R
:= r R : (r

) r

r. So 28
(4.26) q
R
H
R
= ()
R
.
We now see that the following hold:
29
q
P

i
1

[H
P

] does not intersect A in exactly one point. (By assumption.)


30
q
P

i
1

[()
P

] does not intersect A in exactly one point. (By (4.26).)


31
q
P
CS

i
1

[()
P

] does not intersect A in exactly one point. (By (4.24).)


32
q
P
CS

i
1

[()
P
CS

] does not intersect A in exactly one point. (Because i

maps A into P

P
CS

, so 33
A i
1

[(Y)
P

] = A i
1

[(Y)
P
CS

] for all Y.)


34
q
P
CS

i
1

[H
P
CS

] does not intersect A in exactly one point. (Again by (4.26).)


35
But this, according to the denition of P
gen

, implies q P
gen

, a contradiction.
36
We can also show that the almost CS iteration of proper forcings Q

is proper. (We do not really need 37


this fact, as we could allow non-proper iterations in our preparatory forcing, see Section 7.A(4). In some 38
sense, M-completeness replaces properness, so the proof of M-completeness was similar to the usual 39
proof of properness.)
40
Lemma 4.27. Assume that in Denition 4.21, every Q

is (forced to be) proper. Then also each P

is 41
proper.
42
Proof. By induction on we prove that for all < the quotient P

/H

is (forced to be) proper. We 43


use the following facts about properness: 1
(4.28) If P is proper and P forces that Q is proper, then P Q is proper.
9
6
9


r
e
v
i
s
i
o
n
:
2
0
1
1
-
0
5
-
2
8







m
o
d
i
f
i
e
d
:
2
0
1
1
-
0
5
-
2
9


BOREL CONJECTURE AND DUAL BOREL CONJECTURE 33
2
(4.29)
If

P is an iteration of length and if each Q
n
is forced to be proper, then the inverse
limit P

is proper, as are all quotients P

/H
n
.
3
(4.30)
If

P is an iteration of length with cf() > , and if all quotients P

/H

(for <
< ) are forced to be proper, then the direct limit P

is proper, as are all quotients


P

/H

.
If is a successor, then our inductive claim easily follows from the inductive assumption together 4
with (4.28).
5
Let be a limit of countable conality, say = sup
n

n
. Dene an iteration

P

of length with 6
Q

n
P

n+1
/H

n
. (Each Q

n
is proper, by inductive assumption.) There is a natural forcing equivalence 7
between P
CS

and P
CS

, the full CS limit of



P

.
8
Let N H(

) contain

P, P

,

P

, M,

P
M
. Let p P

N. Without loss of generality p P


gen

. So below 9
p we can identify P

with P
CS

and hence with P


CS

; now apply (4.29).


10
The case of uncountable conality is similar, using (4.30) instead.
11
Recall the denition of
n
and and the notion of interpretation Z

(of a code

Z for a null set) from 12


Denition 2.25. We will need the following (local) variant of random preservation:
13
Denition 4.31. Fix a model M, a real r 2

and Q
M
M. Let Q
M
be an M-complete subforcing of Q. 14
We say that Q locally preserves randomness of r over M, if there is a sequence (D
Q
M
k
)
k
in M of open 15
dense subsets of Q
M
such that the following holds: 16
Assume that 17
M thinks that p (p
k
)
k
interprets

Z as Z

(so

Z is a Q
M
-name of a code for a null set and Z

is 18
a code for a null set, both in M),
19
moreover, each p
k
is in D
Q
M
k
(we call such a sequence (p
k
)
k
, or the according interpretation, 20
quick),
21
r is random over M (so there is a minimal c such that Z

c
r).
22
Then there is a q
Q
p
0
forcing that
23
r is random over M[G
M
],
24

Z
c
r.
25
Note that this is trivially satised if r is not random over M.
26
This property is a local variant of strong preservation of randomness (see Denition 2.27). For a 27
variant of this denition, see Section 7.
28
Lemma 4.32. If Q
M
is an ultralaver forcing in M and r a real, then there is an ultralaver forcing 29
Q over
35
Q
M
locally preserving randomness of r over M.
30
If Q
M
is a Janus forcing in M and r a real, then there is a Janus forcing Q over Q
M
(which is in 31
fact equivalent to random forcing) locally preserving randomness of r over M.
32
Proof. Assume that r is random over M (otherwise the claim is vacuous).
33
Ultralaver: This follows directly from Lemma 2.29, setting D
Q
M
n
to be the set of conditions with stem 34
of length at least n.
35
Janus: In this case, the notion of quick interpretations will be trivial, i.e., D
Q
M
k
= Q
M
for all k.
36
According to Lemma 2.43, we can rst nd some countable M

such that r is still random over M

and 37
such that M

thinks that 2

M is countable. In M

the set Q
M
is now a countable Janus forcing; so we can 38
apply Lemma 3.16 to construct a Janus forcing Q
M

over Q
M
which is equivalent to random forcing. In V, 39
let Q be random forcing; since this is a Suslin ccc forcing we know that Q
M

is an M

-complete subforcing 40
of Q. Moreover, as was noted in Lemma 2.28, we know that Q strongly preserves randoms over M

.
41
So assume that (in M) the sequence (p
k
)
k
of Q
M
-conditions interprets

Z as Z

. Since (in M

), Q
M
42
is M-complete in Q
M

, also in M

the Q
M

-name

Z is interpreted as Z

. Let c be such that Z

c
r. So 43
by strong preservation of randoms, we can in V nd some q p
0
forcing that r is random over M

[H
M

] 1
35
Q over Q
M
just means that Q
M
is an M-complete subforcing of Q.
9
6
9


r
e
v
i
s
i
o
n
:
2
0
1
1
-
0
5
-
2
8







m
o
d
i
f
i
e
d
:
2
0
1
1
-
0
5
-
2
9


34 MARTIN GOLDSTERN, JAKOB KELLNER, SAHARON SHELAH, AND WOLFGANG WOHOFSKY
(and therefore also over the subset M[H
M
]), and that

Z
c
r (where

Z can be evaluated in M

[H
M

] or 2
equivalently in M[H
M
]).
3
We will prove the following preservation theorem:
4
Lemma 4.33. Let

P be an almost CS iteration (of length ) over

P
M
, r random over M, and p P
M

. 5
Assume that each P

forces that Q

locally preserves randomness of r over M[H


M

]. Then there is some 6


q p in P

forcing that r is random over M[H


M

].
7
What we will actually need is the following variant:
8
Lemma 4.34. Assume that

P
M
is in M a topped almost CS iteration of length , and we already have some 9
topped partial CS iteration

P over

P
M

0
of length
0
M . Let

r be a P

0
-name of a random real 10
over M[H
M

0
]. Assume that we extend

P to length as an almost CS iteration
36
using forcings Q

which 11
locally preserve the randomness of

r over M, witnessed by a sequence (D


Q
M

k
)
k
. Let p P
M

. Then we can 12
nd a q p in P

forcing that

r is random over M[H


M

].
13
Actually, we will only prove the two previous lemmas under the following additional assumption (which 14
is enough for our application, and saves some unpleasant work). This additional assumption is not really 15
necessary; without it, we could use the method of [GK06] for the proof.
16
Assumption 4.35. For each M , (P
M

forces that) Q
M

is either trivial
37
or adds a new - 17
sequence of ordinals. Note that in the latter case we can assume without loss of generality that 18
_
n
D
Q
M
n
= (and, of course, that the D
Q
M
n
are decreasing).
19
Moreover, we assume that already in M there is a set T such that P

forces: Q

is trivial i 20
T. (So whether Q

is trivial or not does not depend on the generic lter below , it is already 21
decided in the ground model.)
22
The result will follow as a special case of the following lemma, which we prove by induction on . 23
(Note that this is a rened version of the proof of Lemma 4.25 and similar to the proof of the preservation 24
theorem in [Gol93, 5.13].)
25
Denition 4.36. Under the assumptions of Lemma 4.34 and Assumption 4.35, let

Z be a P

-name,
0
26
< , and let p = (p
k
)
k
be a sequence of P

-names of conditions in P

/H

. Let Z

be a P

-name.
27
We say that ( p, Z

) is a quick interpretation of

Z if p interprets

Z as Z

(i.e., P

forces that p
k
forces 28

Zk = Z

k for all k), and moreover:


29
Letting be minimal with Q
M

nontrivial (if such exists): P

forces that the sequence 30


(p
k
())
k
is quick in Q
M

, i.e., p
k
() D
Q
M

k
for all k.
31
It is easy to see that: 32
(4.37) For every name

Z there is a quick interpretation ( p, Z

).
Lemma 4.38. Under the same assumptions as above, let , ,

be as in (4.22) (so in particular we have 33

). 34
Assume that 35
M (= M ) and
0
(so <

),
36
p M is a P
M

-name of a P
M

-condition,
37

Z M is a P
M

-name of a code for null set,


38
Z

M is a P
M

-name of a code for a null set,


39
P
M

forces: p = (p
k
)
k
M is a quick sequence in P
M

/H
M

interpreting

Z as Z

(as in Deni- 40
tion 4.36),
41
P
M

forces: if p H
M

, then p
0
p,
42
q P

forces p H
M

,
1
36
Of course our ocial denition of almost CS iteration assumes that we start the construction at 0, so we modify this denition
in the obvious way.
37
More specically, Q
M

= .
9
6
9


r
e
v
i
s
i
o
n
:
2
0
1
1
-
0
5
-
2
8







m
o
d
i
f
i
e
d
:
2
0
1
1
-
0
5
-
2
9


BOREL CONJECTURE AND DUAL BOREL CONJECTURE 35
q forces that r is randomover M[H
M

], so in particular there is a P

-name c
0
in V for the minimal c 2
with Z

c
r.
3
Then there is a condition q
+
P

, extending q, and forcing the following:


4
p H
M

,
5
r is random over M[H
M

],
6

c
r.
7
We actually claim a slightly stronger version, where instead of Z

and

Z we have nitely many codes 8


for null sets and names of codes for null sets, respectively. We will use this stronger claim as inductive 9
assumption, but for notational simplicity we only prove the weaker version; it is easy to see that the weaker 10
version implies the stronger version.
11
Proof. Nontrivial successor case: = + 1 M.
12
If Q
M

is trivial, there is nothing to do.


13
Assume that is minimal such that Q
M

is a nontrivial forcing notion. Work in V[H

] where q H

. 14
Note that M[H
M

] = M[H
M

]. So r is randomover M[H
M

], and (p
k
())
k
quickly interprets

Z as Z

in Q
M

. 15
Now let q
+
= q, and use the fact that Q

locally preserves randomness to nd q


+
() p
0
().
16
Assume that Q
M

is nontrivial, but that is not minimal. Again work in V[H

]. Let k

be maximal with 17
p
k

H
M

. Consider

Z as a Q
M

-name, and (using (4.37)) nd a quick interpretation Z

of

Z witnessed by 18
a sequence starting with p
k

(). In M[H
M

], Z

is now a P
M

/H
M

-name. Clearly, the sequence (p


k
)
k
is 19
a quick sequence interpreting Z

as Z

. (Use the fact that p


k
forces k

k.) 20
Using the induction hypothesis, we can rst extend q to a condition q

and then (again by our 21


assumption that Q

locally preserves randomness) to a condition q


+
P
+1
.
22
The nontrivial limit case: M unbounded in , i.e.,

= . (This deals with cases (4) and (5) in 23


Denition 4.21. In case (4) we have M, i.e., = ; in case (5) we have M and < .)
24
Let =
0
<
1
< be a sequence of M-ordinals conal in M

= M . We may assume
38
that 25
each Q
M

n
is nontrivial.
26
Let (

Z
n
)
n
be a list of all P
M

-names in M for codes for null sets (starting with our given null set

Z =

Z
0
). 27
Let (E
n
)
n
enumerate all open dense sets of P
M

from M, without loss of generality


39
we can assume that: 28
(4.39) E
n
decides

Z
0
n, . . . ,

Z
n
n.
We write p
k
0
for p
k
, and Z
0,0
for Z

; as mentioned above,

Z =

Z
0
.
29
By induction on n we can now nd a sequence p
n
= (p
k
n
)
k
and P
M

n
-names Z
i,n
for i 0, . . . , n 30
satisfying the following:
31
(1) P
M

n
forces that p
0
n
p
k
n1
whenever p
k
n1
P
M

/H
M

n
.
32
(2) P

n
forces that p
0
n
E
n
. (Clearly E
n
P
M

/H
M

n
is a dense set.)
33
(3) p
n
M is a P
M

n
-name for a quick sequence interpreting (

Z
0
, . . . ,

Z
n
) as (Z
0,n
, . . . , Z
n,n
) (in P
M

/H
M

n
), 34
so Z
i,n
is a P
M

n
-name of a code for a null set, for 0 i n.
35
Note that this implies that the sequence (p
k
n1

n
) is (forced to be) a quick sequence interpreting (Z
0,n
, . . . , Z
n1,n
) 36
as (Z
0,n1
, . . . , Z
n1,n1
) .
37
Using the induction hypothesis, we now dene a sequence (q
n
)
n
of conditions q
n
P

n
and a sequence 38
(c
n
)
n
(where c
n
is a P

n
-name) such that (for n > 0) q
n
extends q
n1
and forces the following:
39
p
0
n1

n
H
M

n
.
40
Therefore, p
0
n
p
0
n1
.
41
r is random over M[H
M

n
].
42
Let c
n
be the least c such that Z
n,n

c
r.
43
Z
i,n

c
i
r for i = 0, . . . , n 1.
44
Now let q =
_
n
q
n
P
CS

. As in Lemma 4.25 it is easy to see that q P


gen

. Moreover, by (4.39) we 45
get that q forces that

Z
i
= lim
n
Z
i,n
. Since each set C
c,r
:= x : x
c
r is closed, this implies that q forces 1

Z
i

c
i
r, in particular

Z =

Z
0

c
0
r.
38
If from some on all Q
M

are trivial, then P


M

= P
M

, so by induction there is nothing to do.


39
well, if we just enumerate a basis of the open sets instead of all of them. . .
9
6
9


r
e
v
i
s
i
o
n
:
2
0
1
1
-
0
5
-
2
8







m
o
d
i
f
i
e
d
:
2
0
1
1
-
0
5
-
2
9


36 MARTIN GOLDSTERN, JAKOB KELLNER, SAHARON SHELAH, AND WOLFGANG WOHOFSKY
2
The trivial cases: In all other cases, M is bounded in , so we already dealt with everything at stage 3

0
sup( M). Note that

0
and
0
used at stage
0
are the same as the current

and .
4
5. Tnr roacrxo coxsratcrrox 5
In this section we describe a -closed preparatory forcing notion R; the generic lter will dene a 6
generic forcing iteration

P, so elements of R will be approximations to such an iteration. In Section 6 we 7
will show that the forcing R P

2
forces BC and dBC.
8
From now on, we assume CH in the ground model.
9
5.A. Alternating iterations, canonical embeddings and the preparatory forcing R. The preparatory 10
forcing R will consist of pairs (M,

P), where M is a countable model and

P M is an iteration of ultralaver 11
and Janus forcings.
12
Denition 5.1. An alternating iteration
40
is a topped partial CS iteration

P of length
2
satisfying the 13
following:
14
Each P

is proper.
41
15
For even, either both Q

and Q
+1
are (forced by the empty condition to be) trivial,
42
or P

16
forces that Q

is an ultralaver forcing adding the generic real


, and P
+1
forces that Q
+1
is a 17
Janus forcing based on

(where

is dened from

as in Lemma 2.22).
18
We will call an even index an ultralaver position and an odd one a Janus position.
19
As in any partial CS iteration, each P

for cf() > (and in particular P

2
) is a direct limit.
20
Recall that in Denition 4.10 we have dened the notion

P
M
canonically embeds into

P for nice 21
candidates M and iterations

P V and

P
M
M. Since our iterations now have length
2
, this means that 22
the canonical embedding works up to and including
43

2
.
23
In the following, we will use pairs x = (M
x
,

P
x
) as conditions in a forcing, where

P
x
is an alternating 24
iteration in the nice candidate M
x
. We will adapt our notation accordingly: Instead of writing M,

P
M
, P
M

25
H
M

(the induced lter), Q


M

, etc, we will write M


x
,

P
x
, P
x

, H
x

, Q
x

, etc. Instead of

P
x
canonically embeds 26
into

P we will say x canonically embeds into

P (which is a more exact notation anyway, since the test 27
whether the embedding is M
x
-complete uses both M
x
and

P
x
, not just

P
x
).
28
The following rephrases Denition 4.10 of a canonical embedding in our new notation, taking into 29
account that: 30
L
D
x
is an M
x
-complete subforcing of L
D
i

D extends

D
x
31
(see (5) of the ultralaver wish list 2.1).
32
Fact 5.2. x = (M
x
,

P
x
) canonically embeds into

P, if (inductively) for all
2
M
x

2
the following 33
holds: 34
Let = + 1 for alpha even (i.e., an ultralaver position). Then either Q
x

is trivial (and Q

can 35
be trivial or not), or we require that (P

forces that) the V[H

]-ultralter system

D used for Q

36
extends the M
x
[H
x

]-ultralter system

D
x
used for Q
x

.
37
Let = +1 for alpha odd (i.e., a Janus position). Then either Q
x

is trivial, or we require that (P

38
forces that) the Janus forcing Q
x

is an M
x
[H
x

]-complete subforcing of the Janus forcing Q

.
39
Let be a limit. Then the canonical extension i

: P
x

is M
x
-complete. (The canonical 40
extension was dened in Denition 4.9.)
41
Fix a suciently large regular cardinal

.
1
40
See Section 7 for possible variants of this denition.
41
This does not seem to be necessary, see Section 7, but it is easy to ensure and might be comforting to some of the readers and/or
authors.
42
For deniteness, let us agree that the trivial forcing is the singleton .
43
This is stronger than to require that the canonical embedding works for every
2
M, even though both P

2
and P
M

2
are
just direct limits; see footnote 32.
9
6
9


r
e
v
i
s
i
o
n
:
2
0
1
1
-
0
5
-
2
8







m
o
d
i
f
i
e
d
:
2
0
1
1
-
0
5
-
2
9


BOREL CONJECTURE AND DUAL BOREL CONJECTURE 37
Denition 5.3. The preparatory forcing R consists of pairs x = (M
x
,

P
x
) such that M
x
H(

) is a nice 2
candidate containing
2
, and

P
x
is in M
x
an alternating iteration (in particular topped and of length
2
). 3
We dene y to be stronger than x (in symbols: y
R
x), if the following holds: either x = y, or:
4
M
x
M
y
and M
x
is countable in M
y
. 5
M
y
thinks that x canonically embeds into

P
y
.
6
We will sometimes write i
x,y
for the canonical embedding (in M
y
) from P
x

2
to P
y

2
.
7
There are several variants of this denition which result in equivalent forcing notions. We will briey 8
come back to this in Section 7. 9
Note that the order on R is transitive. 10
The following is trivial by elementarity:
11
Fact 5.4. Assume that

P is an alternating iteration (in V), that x R canonically embeds into

P, and that 12
N H(

) contains x and

P. Let y = (M
y
,

P
y
) be the ord-collapse of (N,

P). Then y R and y x.
13
This fact will be used, for example, to get from the following Lemma 5.5 to Corollary 5.6.
14
Lemma 5.5. Given x R, there is an alternating iteration

P such that x canonically embeds into

P.
15
Proof. For the proof, we use either of the partial CS constructions introduced in the previous chapter (i.e., 16
an almost CS iteration or an almost FS iteration over

P
x
). The only thing we have to check is that we can 17
indeed choose Q

that satisfy the denition of an alternating iteration (i.e., as ultralaver or Janus forcings) 18
and such that Q
x

is M
x
-complete in Q

.
19
In the ultralaver case we arbitrarily extend

D
x
to an ultralter system

D, which is justied by ultralaver 20
wish list 2.1 item (5).
21
In the Janus case, we take Q

Q
x

(this works by Janus wish list 3.1 item (3). Alternatively, we could 22
extend Q
x

to a random forcing, by Janus wish list item (4).


23
Corollary 5.6. Given x R and an HCON object b H(

) (e.g., a real or an ordinal), there is a y x 24


such that b M
y
. 25
What we will actually need are the following three variants:
26
Lemma 5.7. (1) Given x R there is a -centered alternating iteration

P above x.
27
(2) Given a decreasing sequence x = (x
n
)
n
in R, there is an alternating iteration

P such that each 28
x
n
embeds into

P. Moreover, we can assume that for all Janus positions , the Janus
44
forcing Q

29
is (forced to be) the union of the Q
x
n

, and that for all limits , the forcing P

is the almost FS limit 30


over (x
n
)
n
(as in Corollary 4.19).
31
(3) Let x, y R. Let j
x
be the transitive collapse of M
x
, and dene j
y
analogously. Assume that 32
j
x
[M
x
] = j
y
[M
y
], that j
x
(

P
x
) = j
y
(

P
y
) and that there are
0

1
<
2
such that:
33
M
x

0
= M
y

0
(and thus j
x

0
= j
y

0
).
34
M
x
[
0
,
2
) [
0
,
1
).
35
M
y
[
0
,
2
) [
1
,
2
).
36
Then there is an alternating iteration

P such that both x and y canonically embed into it.
37
Proof. For (1), use an almost FS iteration. We only use the coordinates in M, use the (countable!) Janus 38
forcings Q

Q
x

for all Janus positions M. Ultralaver forcing are -centered anyway, so P

will be 39
-centered, by Lemma 4.17.
40
For (2), use the almost FS iteration over the sequence (x
n
)
n
as in Corollary 4.19, and at Janus positions 41
set Q

to be the union of the Q


x
n

. (By Fact 3.8, Q


x
n

is M
x
n
-complete in Q

, so Corollary 4.19 can be 42


applied here.)
43
For (3), we again use an almost FS construction. This time we start with an almost FS construction over 44
x up to
1
, and then continue with an almost FS construction over y.
45
As above, Fact 5.4 gives us the following consequences:
46
Corollary 5.8. (1) R is -closed. Hence R does not add new HCON objects (and in particular: no 47
new reals).
1
44
If all Q
xn

are trivial, then we may also set Q

to be the trivial forcing, which is formally not a Janus forcing.


9
6
9


r
e
v
i
s
i
o
n
:
2
0
1
1
-
0
5
-
2
8







m
o
d
i
f
i
e
d
:
2
0
1
1
-
0
5
-
2
9


38 MARTIN GOLDSTERN, JAKOB KELLNER, SAHARON SHELAH, AND WOLFGANG WOHOFSKY
(2) R forces that the generic lter G R is -directed, i.e., for every countable subset B of G there is 2
a y G stronger than each element of B.
3
(3) R forces CH. (Since we assume CH in V.)
4
(4) Given a decreasing sequence x = (x
n
)
n
in R and any HCON object b H(

), there is a y R 5
such that 6
y x
n
for all n,
7
M
y
contains b and the sequence x,
8
for all Janus positions , M
y
thinks that the Janus forcing Q
y

is (forced to be) the union of 9


the Q
x
n

,
10
for all limits , M
y
thinks that P
y

is the almost FS limit over (x


n
)
n
(of (P
y

)
<
).
11
Proof. Item (4) directly follows from Lemma 5.7(2) and Fact 5.4. Item (1) is a special case of (4), and (2) 12
and (3) are trivial consequences of (1).
13
Another consequence of Lemma 5.7 is:
14
Lemma 5.9. The forcing notion R is
2
-cc.
15
Proof. Recall that we assume that V (and hence V[G]) satises CH.
16
Assume towards a contradiction that (x
i
: i <
2
) is an antichain. Using CH we may without loss of 17
generality assume that for each i
2
the transitive collapse of (M
x
i
,

P
x
i
) is the same. Set L
i
M
x
i

2
. 18
Using the -lemma we nd some uncountable I
2
such that the L
i
for i I forma -system with root L. 19
Set
0
= sup(L) + 3. Moreover, we may assume sup(L
i
) < min(L
j
\
0
) for all i < j.
20
Now take any i, j I, set x x
i
and y x
j
, and use Lemma 5.7(3). Finally, use Fact 5.4 to nd 21
z x
i
, x
j
.
22
5.B. The generic forcing P

. Let G be R-generic. Obviously G is a


R
-directed system. Using the 23
canonical embeddings, we can construct in V[G] a direct limit P

2
of the directed system G: Formally, 24
we set 25
P

2
(x, p) : x G and p P
x

2
,
and we set (y, q) (x, p) if y
R
x and q is (in y) stronger than i
x,y
(p) (where i
x,y
: P
x

2
P
y

2
is the 26
canonical embedding). Similarly, we dene for each 27
P

:= (x, p) : x G, M
x
and p P
x

with the same order. 28


To summarize: 29
Denition 5.10. For
2
, the direct limit of the P
x

with x G is called P

.
30
Formally, elements of P

2
are dened as pairs (x, p). However, the x does not really contribute any 31
information. In particular:
32
Fact 5.11. (1) Assume that (x, p
x
) and (y, p
y
) are in P

2
, that y x, and that the canonical embedding 33
i
x,y
witnessing y x maps p
x
to p
y
. Then (x, p
x
) =

(y, p
y
).
34
(2) (y, q) is in P

2
stronger than (x, p) i for some (or equivalently: for any) z x, y in G the canoni- 35
cally embedded q is in P
z

2
stronger than the canonically embedded p. The same holds if stronger 36
than is replaced by compatible with or by incompatible with.
37
(3) If (x, p) P

, and if y is such that M


y
= M
x
and

P
y
=

P
x
, then (y, p) =

(x, p).
38
In the following, we will therefore often abuse notation and just write p instead of (x, p) for an element 39
of P

.
40
We can dene a natural restriction map from P

2
to P

, by mapping (x, p) to (x, p). Note that by the 41


fact above, we can assume without loss of generality that M
x
. More exactly: There is a y x in G 42
such that M
y
(according to Corollary 5.6). Then in P

2
we have (x, p) =

(y, p).
43
Fact 5.12. The following is forced by R:
44
P

is completely embedded into P

for <
2
(witnessed by the natural restriction map).
45
If x G, then P
x

is M
x
-completely embedded into P

for
2
(by the identity map p (x, p)).
1
9
6
9


r
e
v
i
s
i
o
n
:
2
0
1
1
-
0
5
-
2
8







m
o
d
i
f
i
e
d
:
2
0
1
1
-
0
5
-
2
9


BOREL CONJECTURE AND DUAL BOREL CONJECTURE 39
If cf() > , then P

is the union of the P

for < .
2
By denition, P

2
is a subset of V.
3
G will always denote an R-generic lter, while the P

2
-generic lter over V[G] will be denoted by H

2
4
(and the induced P

-generic by H

). Recall that for each x G, the map p (x, p) is an M


x
-complete 5
embedding of P
x

2
into P

2
(and of P
x

into P

). This way H

induces an M
x
-generic lter H
x

P
x

.
6
So x R forces that P

is approximated by P
x

. In particular we get:
7
Lemma 5.13. Assume that x R,
2
in M
x
, p P
x

, and is a P
x

-name in M
x
. Then M
x
= p
P
x

( ) 8
i x
R
(x, p)
P

M
x
[H
x

] = ( [H
x

]).
9
Proof. is clear. So assume that ( ) is not forced in M
x
. Then some q
P
x

p forces the negation. Now 10


x forces that (x, q) (x, p) in P

; but the conditions (x, p) and (x, q) force contradictory statements.


11
5.C. The inductive proof of ccc. We will now prove by induction on that P

is (forced to be) ccc and 12


(equivalent to) an alternating iteration. Once we knowthis, we can prove Lemma 5.25, which easily implies 13
all the lemmas in this section. So in particular these lemmas will only be needed to prove ccc and not for 14
anything else (and they will probably not aid the understanding of the construction).
15
In this section, we try to stick to the following notation: R-names are denoted with a tilde underneath 16
(e.g.,

), while P
x

-names or P

-names (for any


2
) are denoted with a dot accent (e.g., ). We use 17
both accents when we deal with R-names for P

-names (e.g.,

).
18
We rst prove a few lemmas that are easy generalizations of the following straightforward observation:
19
Assume that x
R
(

z,

p) P

. In particular, x

z G. We rst strengthen x to some x


1
that decides

z 20
and

p to be z

and p

. Then x
1

, so we can further strengthen x


1
to some y z

. By denition, this 21
means that z

is canonically embedded into



P
y
; so (by Fact 5.11) the P
z

-condition p

can be interpreted as a 22
P
y

-condition as well. So we end up with some y x and a P


y

-condition p

such that y
R
(

z,

p) =

(y, p

).
23
Since Ris -closed, we can immediately generalize this to countably many (R-names for) P

-conditions:
24
Fact 5.14. Assume that x
R

p
n
P

for all n . Then there is a y x and there are p

n
P
y

such that 25
y
R

p
n
=

n
for all n .
26
Recall that more formally we should write: x
R
(

z
n
,

p
n
) P

; and y
R
(

z
n
,

p
n
) =

(y, p

n
).
27
We will need a variant of the previous fact:
28
Lemma 5.15. Assume that P

is forced to be ccc, and assume that x forces (in R) that

r
n
is a P

-name for 29
a real (or an HCON object) for every n . Then there is a y x and there are P
y

-names r

n
in M
y
such 30
that y
R
(
P

r
n
= r

n
) for all n.
31
(Of course, we mean:

r
n
is evaluated by G H

, while r

n
is evaluated by H
y

.)
32
Proof. The proof is an obvious consequence of the previous fact, since names of reals in a ccc forcing can 33
be viewed as a countable sequence of conditions.
34
In more detail: For notational simplicity assume all

r
n
are names for elements of 2

. Working in V, we 35
can nd for each n, m names for a maximal antichain

A
n,m
and for a function

f
n,m
:

A
n,m
2 such that 36
x forces that (P

forces that)

r
n
(m) =

f
n,m
(a) for the unique a

A
n,m
H

. Since P

is ccc, each

A
n,m
is 37
countable, and since R is -closed, it is forced that the sequence

= (

A
n,m
,

f
n,m
)
n,m
is in V.
38
In V, we strengthen x to x
1
to decide

to be some

. We can also assume that

M
x
1
(see 39
Corollary 5.6). Each A

n,m
consists of countably many a such that x
1
forces a P

. Using Fact 5.14 40


iteratively (and again the fact that R is -closed) we get some y x
1
such that each such a is actually an 41
element of P
y

. So in M
y
, we can use (A

n,m
, f

n,m
)
n,m
to construct P
y

-names r

n
in the obvious way.
42
Now assume that y G and that H

is P

-generic over V[G]. Fix any a A

n,m
=

A
n,m
. Since a P
y

, we 43
get a H
y

i a H

. So there is a unique element a of A

n,m
H
y

, and r

n
(m) = f

n,m
(a) =

f
n,m
(a) =

r
n
(m).
44
We will also need the following modication:
45
Lemma 5.16. (Same assumptions as in the previous lemma.) In V[G][H

], set Q

be the union of Q
z

[H
z

] 46
for all z G. In V, assume that x forces that each

r
n
is a name for an element of Q

. Then there is a y x 47
and there is in M
y
a sequence ( r

n
)
n
of P
y

-names for elements of Q


y

such that y forces

r
n
= r

n
for all n.
1
9
6
9


r
e
v
i
s
i
o
n
:
2
0
1
1
-
0
5
-
2
8







m
o
d
i
f
i
e
d
:
2
0
1
1
-
0
5
-
2
9


40 MARTIN GOLDSTERN, JAKOB KELLNER, SAHARON SHELAH, AND WOLFGANG WOHOFSKY
So the dierence to the previous lemma is: We additionally assume that

r
n
is in
_
zG
Q
z

, and we 2
additionally get that r

n
is a name for an element of Q
y

.
3
Proof. Assume x G and work in V[G]. Fix n. P

forces that there is some y


n
G and some P
y
n

-name 4

n
M
y
n
of an element of Q
y
n

such that

r
n
(evaluated by H

) is the same as
n
(evaluated by H
y
n

). Since 5
we assume that P

is ccc, we can nd a countable set Y


n
G of the possible y
n
, i.e., the empty condition 6
of P

forces y
n
Y
n
. (As R is -closed and Y
n
R V, we must have Y
n
V.)
7
So in V, there is (for each n) an R-name

Y
n
for this countable set. Since R is -closed, we can nd 8
some z
0
x deciding each

Y
n
to be some countable set Y

n
R. In particular, for each y Y

n
we know 9
that z
0

R
y G, i.e., z
0

y; so using once again that R is -closed we can nd some z stronger than z


0
10
and all the y
_
n
Y

n
. Let X contain all M
y
such that for some y
_
n
Y

n
, is a P
y

-name for a 11
Q
y

-element. Since z y, each X is actually


45
a P
z

-name for an element of Q


z

.
12
So X is a set of P
z

-names for Q
z

-elements; we can assume that X M


z
. Also, z forces that

r
n
X for 13
all n. Using Lemma 5.15, we can additionally assume that there are names P
z

-name r

n
in M
z
such that z 14
forces that

r
n
= r

n
is forced for each n. By Lemma 5.13, we know that M
z
thinks that P
z

forces that r

n
X. 15
Therefore r

n
is a P
z

-name for a Q
z

-element.
16
We now prove by induction on that P

is equivalent to a ccc alternating iteration:


17
Lemma 5.17. The following holds in V[G] for <
2
:
18
(1) P

is equivalent to an alternating iteration. More formally: There is an iteration (P

, Q

)
<
19
with limit P

that satises the denition of alternating iteration (up to ), and there is a naturally 20
dened dense embedding j

: P

, such that for < we have j

, and the embeddings 21


commute with the restrictions.
46
Each Q

is the union of all Q


x

with x G. For x G with M


x
, 22
the function i
x,
: P
x

that maps p to j

(x, p) is the canonical M


x
-complete embedding.
23
(2) In particular, a P

-generic lter H

can be translated into a P

-generic lter which we call H

24
(and vice versa).
25
(3) P

has a dense subset of size


1
.
26
(4) P

is ccc.
27
(5) P

forces CH.
28
Proof. = 0 is trivial (since P
0
and P

0
both are trivial: P
0
is a singleton, and P

0
consists of pairwise 29
compatible elements).
30
So assume that all items hold for all < .
31
Proof of (1).
32
Ultralaver successor case: Let = +1 with an ultralaver position. Let H

be P

-generic over V[G]. 33


Work in V[G][H

]. By induction, for every x G the canonical embedding j

denes a P
x

-generic lter 34
over M
x
called H
x

.
35
Denition of Q

(and thus of P

): In M
x
[H
x

], the forcing notion Q


x

is dened as L
D
x for some system 36
of ultralters

D
x
in M
x
[H
x

]. Fix some s
<
. If y x in G, then D
y
s
extends D
x
s
. Let D
s
be the union 37
of all D
x
s
with x G. So D
s
is a proper lter. It is even an ultralter: Let r be a P

-name for a real. Using 38


Lemma 5.15, we know that there is some y G and some P
y

-name

r
y
M
y
such that (in V[G][H

]) we 39
have

r
y
[H
y

] = r. So r M
y
[H
y

], hence either r or its complement is in D


y
s
and therefore in D
s
. So all lters 40
in the family

D = (D
s
)
s
< are ultralters.
41
Now work again in V[G]. We set Q

to be the P

-name for L
D
. (Note that P

forces that Q

literally is 42
the union of the Q
x

[H
x

] for x G, again by Lemma 5.15.)


43
Denition of j

: Let (x, p) be in P

. If p P
x

, then we set j

(x, p) = j

(x, p), i.e., j

will extend j

. If 44
p = (p, p()) is in P
x

but not in P
x

, we set j

(x, p) = (r, s) P

where r = j

(x, p) and s is the 45


(P

-name for) p() as evaluated in M


x
[H
x

]. From Q

=
_
xG
Q
x

[H
x

] we conclude that this embedding is 46


dense. 1
45
Here we use two consequences of z y: Every P
y

-name in M
y
can be canonically interpreted as a P
z

-name in M
z
, and Q
y

is
(forced to be) a subset of Q
z

.
46
I.e., j

(x, p) = j

(x, p) = j

(x, p).
9
6
9


r
e
v
i
s
i
o
n
:
2
0
1
1
-
0
5
-
2
8







m
o
d
i
f
i
e
d
:
2
0
1
1
-
0
5
-
2
9


BOREL CONJECTURE AND DUAL BOREL CONJECTURE 41
The canonical embedding: By induction we know that i
x,
which maps p P
x

to j

(x, p) is (the restric- 2


tion to P
x

of) the canonical embedding of x into P

2
. So we have to extend the canonical embedding to 3
i
x,
: P
x

. By denition of canonical embedding, i


x,
maps p P
x

to the pair (i
x,
(p), p()). This 4
is the same as j

(x, p). We already know that D


x
s
is (forced to be) an M
x
[H
x

]-ultralter that is extended 5


by D
s
.
6
Janus successor case: This is similar, but simpler than the previous case: Here, Q

is just dened as 7
the union of all Q
x

[H
x

] for x G. We will show below that this union satises the ccc; just as in Fact 3.8, 8
it is then easy to see that this union is again a Janus forcing.
9
In particular, Q

consists of hereditarily countable objects (since it is the union of Janus forcings, which 10
by denition consist of hereditarily countable objects). So since P

forces CH, Q

is forced to have size


1
. 11
Also note that since all Janus forcings involved are separative, the union (which is a limit of an incom- 12
patibility-preserving directed system) is trivially separative as well.
13
Limit case: Let be a limit ordinal. 14
Denition of P

and j

: First we dene j

: P

P
CS

: For each (x, p) P

, let j

(x, p) P

be 15
the union of all j

(x, p) (for M
x
). (Note that
1
<
2
implies that j

1
(x, p
1
) is restriction of 16
j

2
(x, p
2
), so this union is indeed an element of P
CS

.)
17
P

is the set of all q p, where p j

[P

], q P

for some < , and q p.


18
It is easy to check that P

actually is a partial countable support limit, and that j

is dense. We will 19
show below that P

satises the ccc, so in particular it is proper.


20
The canonical embedding: To see that i
x,
is the (restriction of the) canonical embedding, we just have 21
to check that i
x,
is M
x
-complete. This is the case since P

is the direct limit of all P


y

for y G (without 22
loss of generality y x), and each i
x,y
is M
x
-complete (see Fact 5.12).
23
Proof of (3).
24
Recall that we assume CH in the ground model.
25
The successor case, = + 1, follows easily from (3)(5) for P

(since P

forces that Q

has size 26
2

0
=
1
=
V
1
).
27
If cf() > , then P

=
_
<
P

, so the proof is easy.


28
So let cf() = . The following straightforward argument works for any ccc partial CS iteration where 29
are all iterands Q

are of size
1
.
30
For notational simplicity we assume
P


1
for all < (this is justied by inductive assump- 31
tion (5)). By induction, we can assume that for all < there is a dense P

of size
1
and that every 32
P

is ccc. For each p P

and all dom(p) we can nd a maximal antichain A


p

such that each 33


element a A
p

decides the value of p(), say a


P

p() =
p

(a). Writing
47
p q if p q and q p, the 34
map p (A
p

,
p

)
dom(p)
is 1-1 modulo . Since each A
p

is countable, there are only


1
many possible 35
values, therefore there are only
1
many -equivalence classes. Any set of representatives will be dense.
36
Alternatively, we can prove (3) directly for P

. I.e., we can nd a

-dense subset P

of cardinality 37

1
. Note that a conditions (x, p) P

essentially depends only on p (cf. Fact 5.11). More specically, given 38


(x, p) we can transitively
48
collapse x above , resulting in a =

-equivalent condition (x

, p

). Since 39
=
1
, there are only
1

0
= 2

0
many such candidates x

and since each x

is countable and p

, 40
there are only 2

0
many pairs (x

, p

).
41
Proof of (4).
42
Ultralaver successor case: Let = + 1 with an ultralaver position. We already know that P

= 43
P

where Q

is an ultralaver forcing, which in particular is ccc, so by induction P

is ccc.
44
Janus successor case: As above it suces to show that Q

, the union of the Janus forcings Q


x

[H
x

] for 45
x G, is (forced to be) ccc.
1
47
Since is separative, p q i p =

q, but this fact is not used here.


48
In more detail: We dene a function f : M
x
V by induction as follows: If M
x
+ 1 or if =
2
, then f () = .
Otherwise, if M
x
Ord, then f () is the smallest ordinal above f []. If a M
x
\ Ord, then f (a) = f (b) : b a M
x
. It is
easy to see that f is an isomorphism from M
x
to f [M
x
] := M
x

and that M
x

is a candidate. Moreover, the ordinals that occur in M


x

are subsets of +
1
together with the interval [
2
,
2
+
1
]; i.e., there are
1
many ordinals that can possibly occur in M
x

, and
therefore there are 2

0
many possible such candidates. Moreover, setting p

f (p), it is easy to check that (x, p) =

(x, p) (similarly
to Fact 5.11).
9
6
9


r
e
v
i
s
i
o
n
:
2
0
1
1
-
0
5
-
2
8







m
o
d
i
f
i
e
d
:
2
0
1
1
-
0
5
-
2
9


42 MARTIN GOLDSTERN, JAKOB KELLNER, SAHARON SHELAH, AND WOLFGANG WOHOFSKY
Assume towards a contradiction that this is not the case, i.e., that we have an uncountable antichain 2
in Q

. We already know that Q

has size
1
and therefore the uncountable antichain has size
1
. So, 3
working in V, we assume towards a contradiction that 4
(5.18) x
0

R
p
0

P

a
i
: i
1
is a maximal (uncountable) antichain in Q

.
We construct by induction on n a decreasing sequence of conditions such that x
n+1
satises the 5
following:
6
(i) For all i
1
M
x
n
there is (in M
x
n+1
) a P
x
n+1

-name a

i
for a Q
x
n+1

-condition such that 7


x
n+1

R
p
0

P

a
i
= a

i
.
Why can we get that? Just use Lemma 5.16.
8
(ii) If is in M
x
n
a P
x
n

-name for an element of Q


x
n

, then there is k

()
1
such that 9
x
n+1

R
p
0

P

(i < k

())

a
i
j
P

.
Also, all these k

() are in M
x
n+1
. 10
Why can we get that? First note that x
n
p
0
(i
1
)

a
i
j . Since P

is ccc, x
n
forces that there 11
is some bound

k() for i. So it suces that x


n+1
determines

k() to be k

() (for all the countably 12


many ).
13
Set


1

_
n
M
x
n
. By Corollary 5.8(4), there is some y such that
14
y x
n
for all n ,
15
(x
n
)
n
and ( a

i
)
i
are in M
y
,
16
(M
y
thinks that) P
y

forces that Q
y

is the union of Q
x
n

.
17
Let G be R-generic (over V) containing y, and let H

be P

-generic (over V[G]) containing p


0
.
18
Set A

i
[H
y

] : i <

. Note that A

is in M
y
[H
y

]. We claim 19
(5.19) A

Q
y

[H
y

] is predense.
Pick any q
0
Q
y

. So there is some n and some which is in M


x
n
a P
x
n

-name of a Q
x
n

-condition, such 20
that q
0
= [H
x
n

]. By (ii) above, x
n+1
and therefore y forces (in R) that for some i < k

() (and therefore 21
some i <

) the condition p
0
forces the following (in P

):
22
The conditions

a
i
and are compatible in Q

. Also,

a
i
= a

i
and both are in Q
y

, and Q
y

23
is an incompatibility-preserving subforcing of Q

. Therefore M
y
[H
y

] thinks that a

i
and 24
are compatible.
25
This proves (5.19).
26
Since Q
y

[H
y

] is M
y
[H
y

]-complete in Q

[H

], and since A

M
y
[H
y
], this implies (as a

i
[H
y

] =

a
i
[G 27
H

] for all i <

) that

a
i
[G H

] : i <

already is predense, a contradiction to (5.18).


28
Limit case: We work with P

, which by denition only contains HCON objects.


29
Assume towards a contradiction that P

has an uncountable antichain. We already know that P

has a 30
dense subset of size
1
(modulo =

), so the antichain has size


1
.
31
Again, work in V. We assume towards a contradiction that 32
(5.20) x
0

R

a
i
: i
1
is a maximal (uncountable) antichain in P

.
So each

a
i
is an R-name for an HCON object (x, p) in V.
33
To lighten the notation we will abbreviate elements (x, p) P

by p; this is justied by Fact 5.11.


34
Fix any HCON object p and < . We will now dene the (R P

)-names

(, p) and

r(, p): Let G 35


be R-generic and containing x
0
, and H

be P

-generic. Let R be the quotient P

/H

. If p is not in R, set 36

(, p) =

r(, p) = 0. Otherwise, let

(, p) be the minimal i such that

a
i
R and

a
i
and p are compatible 37
(in R), and set

r(, p) R to be a witness of this compatibility. Since P

is (forced to be) ccc, we can 38


nd (in V[G]) a countable set

(, p)
1
containing all possibilities for

(, p) and similarly

X
r
(, p) 39
consisting of HCON objects for

r(, p).
40
To summarize: For every < and every HCON object p, we can dene (in V) the R-names

(, p) 41
and

X
r
(, p) such that 1
(5.21) x
0

R

P

_
p P

/H

(i

(, p)) (r

X
r
(, p)) r
P

/H

p,

a
i
_
.
9
6
9


r
e
v
i
s
i
o
n
:
2
0
1
1
-
0
5
-
2
8







m
o
d
i
f
i
e
d
:
2
0
1
1
-
0
5
-
2
9


BOREL CONJECTURE AND DUAL BOREL CONJECTURE 43
Similarly to the Janus successor case, we dene by induction on n a decreasing sequence of con- 2
ditions such that x
n+1
satises the following: For all M
x
n
and p P
x
n

, x
n+1
decides

(, p) and 3

X
r
(, p) to be some X

(, p) and X
r
(, p). For all i
1
M
x
n
, x
n+1
decides

a
i
to be some a

i
P
x
n+1

. 4
Moreover, each such X

and X
r
is in M
x
n+1
, and every r X
r
(, p) is in P
x
n+1

. (For this, we just use 5


Fact 5.14 and Lemma 5.15.)
6
Set


1

_
n
M
x
n
, and set A

i
: i

. By Corollary 5.8(4), there is some y such that


7
y x
n
for all n ,
8
x (x
n
)
n
and A

are in M
y
,
9
(M
y
thinks that) P
y

is dened as the almost FS limit over x.


10
We claim that y forces 11
(5.22) A

is predense in P
y

.
Then P
y

is M
y
-completely embedded into P

, and since A

M
y
(and since

a
i
= a

i
for all i

) we get 12
that

a
i
: i

is predense, a contradiction to (5.20).


13
So it remains to show (5.22). Let G be R-generic containing y. Let r be a condition in P
y

; we will nd 14
i <

such that r is compatible with a

i
. Since P
y

is the almost FS limit over x, there is some n and 15


M
x
n
such that r has the form q p with p in P
x
n

, q P
y

and q p.
16
Now let H

be P

-generic containing q. Work in V[G][H

]. Since q p, we get p P

/H

. Let

17
be the evaluation by G H

of

(, p), and let r

be the evaluation of

r(, p). Note that

<

and r

P
y

. 18
So we know that a

and p are compatible in P

/H

witnessed by r

. Find q

forcing r

/H

p, a

. 19
We may nd q

q. Now q

witnesses that q p and a

are compatible in P
y

.
20
Proof of (5).
21
This follows from (3) and (4).
22
5.D. The generic alternating iteration

P. In Lemma 5.17 we have seen:
23
Corollary 5.23. Let G be R-generic. Then we can construct
49
(in V[G]) an alternating iteration

P such 24
that the following holds:
25


P is ccc. 26
If x G, then x canonically embeds into

P. (In particular, a P

2
-generic lter H

2
induces a 27
P
x

2
-generic lter over M
x
, called H
x

2
.)
28
Each Q

is the union of all Q


x

[H
x

] with x G.
29
P

2
is equivalent to the direct limit P

2
of G: There is a dense embedding j : P

2
P

2
, and for 30
each x G the function p j(x, p) is the canonical embedding.
31
Lemma 5.24. Let x R. Then R forces the following: x G i x canonically embeds into

P

2
.
32
Proof. If x G, then we already know that x canonically embeds into

P.
33
So assume (towards a contradiction) that y forces that x embeds, and that x y (i.e., y x G). Work 34
in V[G] where y G. Both x (by assumption) and y G canonically embed into

P. Let N be an elementary 35
submodel of H
V[G]
(

) containing x, y,

P; let z = (M
z
,

P
z
) be the ord-collapse of (N,

P). Then z V (as R 36
is -closed) and z R, and (by elementarity) z x, y. This shows that x j
R
y, i.e., y cannot force x G, a 37
contradiction. 38
Using ccc, we can nowprove a lemma that is in fact stronger than the lemmas in the previous section 5.C:
39
Lemma 5.25. The following is forced by R: Let N H
V[G]
(

) be countable, and let y be the ord-collapse 40


of (N,

P). Then y G. Moreover, if x G N, then y x.
41
Proof. Work in V[G] with x G. Pick an elementary submodel N containing x and

P. Let y be the ord- 42
collapse of (N,

P) via a collapsing map k. As above, it is clear that y R and y x. To show y G, it 43
is (by the previous lemma) enough to show that y canonically embeds. We claim that k
1
is the canonical 44
embedding of y into

P. The crucial point is to show M
y
-completeness. Let B M
y
be a maximal antichain 45
of P
y

2
, say B = k(A) where A N is a maximal antichain of P

2
. So (by ccc) A is countable, hence A N. 46
So not only A = k
1
(B) but even A = k
1
[B]. Hence k
1
is an M
y
-complete embedding.
1
49
in an absolute way: Given G, we rst dene P

2
to be the direct limit of G, and then inductively construct P

from P

2
.
9
6
9


r
e
v
i
s
i
o
n
:
2
0
1
1
-
0
5
-
2
8







m
o
d
i
f
i
e
d
:
2
0
1
1
-
0
5
-
2
9


44 MARTIN GOLDSTERN, JAKOB KELLNER, SAHARON SHELAH, AND WOLFGANG WOHOFSKY
Remark 5.26. Using this lemma we can easily reprove the countable chain condition of P

2
and in fact all 2
other lemmas in section 5.C. We will do this for Fact 5.14 in Corollary 6.4.
3
6. Tnr raoor or BC+dBC 4
We rst prove that no uncountable X in V will be smz or sm in the extension.
50
Then we show how to 5
modify the argument to work for all uncountable sets in V[G H].
6
6.A. BC+dBC for ground model sets. A schematic diagramillustrating the arguments in this section can 7
be found in Figure 2 on page 8.
8
Lemma 6.1. Let X V be an uncountable set of reals. Then R P

2
forces that X is not smz.
9
Proof.
10
(1) Fix any even <
2
(i.e., an ultralaver position) in our iteration. The ultralaver forcing Q

adds a 11
(canonically dened code for a) closed null set

F constructed from the ultralaver real

. (Recall 12
the ultralaver wish list 2.1, item (2) and item (3). In the following, when we consider various 13
forcings Q

, Q

, Q
x

, we treat

F not as an actual name, but rather as a denition which depends on 14
the forcing used.)
15
(2) According to Theorem 1.2, it is enough to show that X +

F is non-null in the R P

2
-extension, or 16
equivalently, in every R P

-extension ( < <


2
). So assume towards a contradiction that there 17
is a > and an R P

-name

Z of a (code for a) Borel null set such that some (x, p) R P

2
18
forces that X +

F

Z.
19
(3) Using the dense embedding j

2
: P

2
P

2
, we may replace (x, p) by a condition (x, p

) RP

2
. 20
According to Fact 5.14 (recall that we now know that P

2
satises ccc) and Lemma 5.15 we can 21
assume that p

is already a P
x

-condition p
x
and that

Z is (forced by x to be the same as) a P


x

-name 22

Z
x
in M
x
. 23
(4) We construct (in V) an iteration

P in the following way:
24
(a) Up to , we take an arbitrary alternating iteration into which x embeds. In particular, P

will 25
be proper and hence forces that X is still uncountable.
26
(b) Let Q

be any ultralaver forcing (over Q


x

in case M
x
). So according to item (2) of the 27
ultralaver wish list 2.1, we know that Q

forces that X +

F is not null.
28
Therefore we can pick (in V[H
+1
]) some r in X +

F which is random over (the countable 29
model) M
x
[H
x
+1
], where H
x
+1
is induced by H
+1
.
30
(c) In the rest of the construction, we preserve randomness of r over M
x
[H
x

] for each
2
. We 31
can do this using an almost CS iteration over x where at each Janus position we use a random 32
version of Janus forcing and at each ultralaver position we use a suitable ultralaver forcing; 33
this is possible by Lemma 4.32. By Lemma 4.34, this iteration will preserve the randomness 34
of r. 35
(d) So we get

P over x (with canonical embedding i
x
) and q
P

2
i
x
(p
x
) such that q forces (in 36
P

) that r is random over M


x
[H
x

], in particular that r

Z
x
.
37
We now pick a countable N H(

) containing everything and ord-collapse (N,



P) to y x. (See 38
Fact 5.4.) Set X
y
X M
y
(the image of X under the collapse). By elementarity, M
y
thinks that 39
(a)(d) above holds for

P
y
and that X
y
is uncountable. Note that X
y
X.
40
(5) This gives a contradiction in the obvious way: Let G be R-generic over V and contain y, and let H

41
be P

-generic over V[G] and contain q. So M


y
[H
y

] thinks that r

Z
x
(which is absolute) and 42
that r = x + f for some x X
y
X and f F (actually even in F as evaluated in M
y
[H
y
+1
]). So 43
in V[G][H

], r is the sum of an element of X and an element of F. So (y, q) (x, p

) forces that 44
r X +

F \

Z, a contradiction to (2).
45
Of course, we need this result not just for ground model sets X, but for R P

2
-names

X = (

x
i
: i
1
) 46
of uncountable sets. It is easy to see that it is enough to deal with R P

-names for (all) <


2
. So given 47

X, we can (in the proof) pick such that

X is actually an RP

-name. We can try to repeat the same proof; 1


50
Note that for this weak version, it would be enough to produce a generic iteration of length 2 only, i.e., Q
0
Q
1
, where Q
0
is an
ultralaver forcing and Q
1
an corresponding Janus forcing.
9
6
9


r
e
v
i
s
i
o
n
:
2
0
1
1
-
0
5
-
2
8







m
o
d
i
f
i
e
d
:
2
0
1
1
-
0
5
-
2
9


BOREL CONJECTURE AND DUAL BOREL CONJECTURE 45
however, the problem is the following: When constructing

P in (4), it is not clear how to simultaneously 2
make all the uncountably many names (

x
i
) into

P-names in a suciently absolute way. In other words: 3
It is not clear how to end up with some M
y
and

X
y
uncountable in M
y
such that it is guaranteed that

X
y
4
(evaluated in M
y
[H
y

]) will be a subset of

X (evaluated in V[G][H

]). We will solve this problem in the 5


next section by factoring R.
6
Let us now give the proof of the corresponding weak version of dBC:
7
Lemma 6.2. Let X V be an uncountable set of reals. Then R P

2
forces that X is not strongly meager.
8
Proof. The proof is parallel to the previous one:
9
(1) Fix any even <
2
(i.e., an ultralaver position) in our iteration. The Janus forcing Q
+1
adds a 10
(canonically dened code for a) null set

Z

. (Recall the Janus wish list 3.1, item (1).)


11
(2) According to (1.8), it is enough to show that X +

Z

= 2

in the R P

2
-extension, or equivalently, 12
in every R P

-extension ( < <


2
). (For every real r, the statement r X +

Z

, i.e., 13
(x X) x + r

Z

, is absolute.) So assume towards a contradiction that there is a > and an 14


R P

-name

r of a real such that some (x, p) R P

2
forces that

r X +

Z

.
15
(3) Again, we can assume that

r is a P
x

-name r
x
in M
x
.
16
(4) We construct (in V) an iteration

P in the following way:
17
(a) Up to , we take an arbitrary alternating iteration into which x embeds. In particular, P

again 18
forces that X is still uncountable. 19
(b1) Let Q

be any ultralaver forcing (over Q


x

). Then Q

forces that X is not thin (see item (4) of 20


the ultralaver wish list 2.1).
21
(b2) Let Q
+1
be a countable Janus forcing. So Q
+1
forces X +

Z

= 2

. (See Janus wish 22


list 3.1(1).)
23
(c) We continue the iteration in a -centered way. I.e., we use an almost FS iteration over x 24
of ultralaver forcings and countable Janus forcings, using trivial Q

for all M
x
; see 25
Lemma 4.17. 26
(d) So P

still forces that X +



Z

= 2

, and in particular that r


x
X +

Z

. (See Janus wish list 3.1 27


item (1)).
28
Again, by collapsing some N as in the previous proof, we get y x and X
y
X.
29
(5) This again gives the obvious contradiction: Let G be R-generic over V and contain y, and let H

30
be P

-generic over V[G] and contain p. So M


y
[H
y

] thinks that r = x + z for some x X


y
X and 31
z Z

(this time, Z

is evaluated in M
y
[H
y

]), contradicting (2).


32
6.B. A factor lemma. We can restrict R to any

<
2
in the obvious way: Conditions are pairs x = 33
(M
x
,

P
x
) of nice candidates M
x
(containing

) and alternating iterations



P
x
, but now M
x
thinks that

P
x
has 34
length

(and not
2
). We call this variant R

.
35
Note that all results of Section 5 about R are still true for R

. In particular, whenever G R

is 36
generic, it will dene a direct limit (which we call P

), and an alternating iteration of length

(called P

); 37
again we will have that x G i x canonically embeds into

P

.
38
There is a natural projection map from R (more exactly: from the dense subset of those x which satisfy 39

M
x
) into R

, mapping x = (M
x
,

P
x
) to x

(M
x
,

P
x

). (It is obvious that this projection is 40


dense and preserves .)
41
There is also a natural embedding from R

to R: We can just continue an alternating iteration of 42


length

by appending trivial forcings.


43
is complete: It preserves and . (Assume that z (x), (y). Then z

x, y.) Also, the projection 44


is a reduction: If y x

in R

, then let M
z
be a model containg both x and y. In M
z
, we can rst 45
construct an alternating iteration of length

over y (using almost FS over y, or almost CS this does 46


not matter here). We then continue this iteration

P
z
using almost FS or almost CS over x. So x and y both 47
embed into

P
z
, hence z = (M
z
,

P
z
) x, y.
48
So according to the general factor lemma of forcing theory, we know that R is forcing equivalent to 49
R

(R/R

), where R/R

is the quotient of R and R

, i.e., the (R

-name for the) set of x R 50


which are compatible (in R) with all (y) for y G

(the generic lter for R

), or equivalently, the set 51


of x R such that x

. So Lemma 5.24 (relativized to R

) implies: 1
(6.3) R/R

is the set of x R that canonically embed (up to

) into P

.
9
6
9


r
e
v
i
s
i
o
n
:
2
0
1
1
-
0
5
-
2
8







m
o
d
i
f
i
e
d
:
2
0
1
1
-
0
5
-
2
9


46 MARTIN GOLDSTERN, JAKOB KELLNER, SAHARON SHELAH, AND WOLFGANG WOHOFSKY
Setup. Fix some

<
2
of uncountable conality.
51
Let G

be R

-generic over V and work in 2


V

V[G

]. Set

P

= (P

)
<
, the generic alternating iteration added by R

. Let R

be the quotient 3
R/R

.
4
We claim that R

satises (in V

) all the properties that we proved in Section 5 for R (in V), with the 5
obvious modications. In particular:
6
(A)

is
2
-cc, since it is the quotient of an
2
-cc forcing.
7
(B)

does not add new reals (and more generally, no new HCON objects), since it is the quotient of a 8
-closed forcing.
52
9
(C)

Let G

be R

-generic over V

. Then G

is R-generic over V, and therefore Corollary 5.23 holds 10


for G

. (Note that P

2
and then P

2
is constructed from G

.) Moreover, it is easy to see


53
that

P 11
starts with

P

. 12
(D)

In particular, we get a variant of Lemma 5.25: The following is forced by R

: Let N H
V[G

]
(

) 13
be countable, and let y be the ord-collapse of (N,

P). Then y G

. Moreover: If x G

N, then 14
y x.
15
We can use the last item to prove the R

-version of Fact 5.14 (which we could of course also reprove 16


directly):
17
Corollary 6.4. Assume that x R

forces that p P

2
. Then there is a y x and a p
y
P
y

2
such that y 18
forces p
y
=

p.
19
Proof. Let G

contain x. In V[G

], pick an elementary submodel N containing x, p,



P and let (M
z
,

P
z
, p
z
) 20
be the ord-collapse of (N,

P, p). Then z G

. This whole situation is forced by some y z x G

. So y 21
and p
y
is as required, where p
y
P
y

2
is the canonical image of p
z
.
22
We nowclaimthat RP

2
forces BC+dBC. We knowthat Ris forcing equivalent to R

. Obviously 23
we have 24
R P

2
= R

,
2
(where P

,
2
is the quotient of P

2
and P

). Note that P

is already determined by R

, so R

is 25
(forced by R

to be) a product R

= P

.
26
But note that this is not the same P

, where we evaluate the denition of R

in the P

-extension 27
of V[G

]: We would get new candidates and therefore new conditions in R

after forcing with P

. In 28
other words, we can unfortunately not just argue as follows:
29
Wrong argument. R P

2
is the same as (R

) (R

,
2
); so given an R P

2
-name X of a 30
set of reals of size
1
, we can choose

large enough so that X is an (R

)-name. Then, working in 31


the (R

)-extension, we just apply Lemmas 6.1 and 6.2.


32
So what do we do instead? Assume that

X =

i
: i
1
is an R P

2
-name for a set of reals of 33
size
1
. So there is a <
2
such that

X is added by R P

(using
2
-cc of R). In the R-extension, 34
P

is ccc, therefore we can assume that each

i
is a system of countably many countable antichains

A
m
i
35
of P

, together with functions

f
m
i
:

A
m
i
0, 1. For the following argument, we prefer to work with the 36
equivalent P

instead of P

. We can assume that each of the sequences B


i
(

A
m
i
,

f
m
i
)
m
is an element 37
of V (since P

is a subset of V and since R is -closed). So each B


i
is decided by a maximal antichain Z
i
38
of R. Since R is
2
-cc, these
1
many antichains all are contained in some R

with

.
39
From now on, we work in the R

-extension V

. So in V

we have the following situation: Each


i
is 40
a very absolute R

-name (or equivalently, R

-name), in fact they are already determined by 41


antichains that are in P

and do not depend on R

. So we can interpret them as P

-names.
42
Note that the
i
are forced (by R

) to be pairwise dierent, and therefore already by P

.
1
51
Probably the conality is completely irrelevant, but the picture is clearer this way.
52
It is easy to see that R

is even -closed, by relativizing the proof for R, but we will not need this.
53
Let P

be the direct limit of G

(for

), and P

the direct limit of G

. The function k

: P

that maps (x, p) to


((x), p) preserves and and is surjective modulo =

, see Fact 5.11(3). So it is clear that dening



P

by induction from P

2
yields
the same result as dening

P from P

2
.
9
6
9


r
e
v
i
s
i
o
n
:
2
0
1
1
-
0
5
-
2
8







m
o
d
i
f
i
e
d
:
2
0
1
1
-
0
5
-
2
9


BOREL CONJECTURE AND DUAL BOREL CONJECTURE 47
So now we can just repeat the proofs of BC and dBC of Section 6.A, with the following modications 2
(the modications are the same for both proofs): We x the uncountable set

X, choose the according

as 3
above, and work in the R

extension V

.
4
(1) Instead of any ultralaver position <
2
, we obviously have to choose an

.
5
(2) No change here (of course we now have an R

-name).
6
(3) Here, we use Corollary 6.4.
7
(4) The iteration obviously has to start with

P

(which is ccc), the rest is the same. Since



P starts with 8

, the ord-collapse y of (N,



P) is in R

. Now comes the relevant point of the factoring: Since 9

X consists of absolute P

-names, we know that if y is in the R

-generic lter G

, and if H is 10
P

2
-generic over V[G

], then

X
y
[H] =

X[H] M
y


X[H]. So y thinks that

X
y
is a name for an 11
uncountable set, and we know that this P
y

2
-name will be evaluated to a subset of

X.
12
(5) No change here.
13
7. A woao ox vxarxxrs or rnr orrrxrrroxs 14
The following is not needed for understanding the paper, we just briey comment on alternative ways 15
some notions could be dened. 16
7.A. Regarding alternating iterations. We call the set of
2
such that Q

is (forced to be) nontriv- 17


ial the true domain of

P (we use this notation in this remark only). Obviously

P is naturally isomorphic 18
to an iteration whose length is the order type of its true domain. In Denitions 5.1 and 5.3, we could have 19
imposed the the following additional requirements. All these variants lead to equivalent forcing notions.
20
(1) M
x
is (an ord-collapse of) an elementary submodel of H(

). 21
This is equivalent, as conditions coming from elementary submodels are dense in our R, by 22
Fact 5.4. 23
While this denition looks much simpler and therefore nicer (we could replace ord-transitive mod- 24
els by the better understood elementary models), it would not make things easier and just hides 25
the point of the construction: For example, we use models M
x
that are (an ord-collapse of) an 26
elementary submodel of H
V

) for some forcing extension V

of V.
27
(2) Require that (M
x
thinks that) the true domain of

P
x
is
2
. 28
This is equivalent for the same reason as (1) (and this requirement is compatible with (1)). 29
This denition would allow to drop the trivial option from the denition, but it would make the 30
dBC argument more cumbersome, as an iteration with uncountably many nontrivial iterands Q

is 31
not -centered. 32
(3) Alternatively, require that (M
x
thinks that) the true domain of

P
x
is countable. 33
Again, equivalence can be seen as in (1), again (3) is compatible with (1) but obviously not with (2). 34
This requirement would not make the denition easier, so there is no reason to adopt it. It would 35
have the slight inconvenience that instead of using ord-collapses as in Fact 5.4, we would have to 36
put another model on top to make the iteration countable. Also, it would have the (purely aesthetic) 37
disadvantage that the generic iteration itself does not satisfy this requirement.
38
(4) Also, we could have dropped the requirement that the iteration is proper. It is never directly used, 39
and densely

P is proper anyway. (E.g., in Lemma 6.1(4a), we would just construct

P up to to 40
be proper or even ccc, so that X remains uncountable.)
41
7.B. Regarding almost CS iterations and separative iterands. Recall that in Denition 4.6 we required 42
that each iterand Q

in a partial CS iteration is separative. This implies the property (actually: the three 43
equivalent properties) from Fact 4.8. Let us call this property suitability for now. Suitability is a prop- 44
erty of the limit P

of

P. Suitability always holds for nite support iterations and for countable support 45
iterations. However, if we do not assume that each Q

is separative, then suitability may fail for par- 46


tial CS iterations. We could drop the separativity assumption, and instead add suitability as an additional 47
requirement to the denition of partial CS limit.
48
The disadvantage of this approach is that we would have to check in all constructions of partial CS 49
iterations that suitability is indeed satised (which is straightforward but surprisingly is rather cumbersome, 50
in particular in the case of the almost CS iteration, where an additional, stronger form of suitability has to 51
be introduced).
1
9
6
9


r
e
v
i
s
i
o
n
:
2
0
1
1
-
0
5
-
2
8







m
o
d
i
f
i
e
d
:
2
0
1
1
-
0
5
-
2
9


48 MARTIN GOLDSTERN, JAKOB KELLNER, SAHARON SHELAH, AND WOLFGANG WOHOFSKY
In contrast, the disadvantage of assuming that Q

is separative is minimal and purely cosmetic: It is 2


well known that every quasiorder Q can be made into a separative one which is forcing equivalent to the 3
original Q (e.g., by just redening the order to be

Q
).
4
7.C. Regarding preservation of random and quick sequences. Recall Denition 4.31 of local preser- 5
vation of random reals and Lemma 4.32. 6
In some respect the dense sets D
n
are unnecessary. For ultralaver forcing L
D
, the notion of a quick 7
sequence refers to the sets D
n
of conditions with stem of length at least n.
8
We could dene a new partial order on L
D
as follows: 9
q

p (q = p) or (q p and the stem of q is strictly longer than the stem of p)


Then (L
D
, ) and (L
D
,

) are forcing equivalent, and any

-interpretation of a new real will automatically 10


be quick.
11
Note however that (L
D
,

) is nownot separative any more. Therefore we chose not to take this approach, 12
since losing separativity causes technical inconvenience, as described in 7.B.
13
Rrrrarxcrs 14
[BJ95] Tomek Bartoszy nski and Haim Judah. Set theory. A K Peters Ltd., Wellesley, MA, 1995. On the structure of the real line.
15
[Bor19] E. Borel. Sur la classication des ensembles de mesure nulle. Bull. Soc. Math. France, 47:97125, 1919.
16
[BS03] Tomek Bartoszynski and Saharon Shelah. Strongly meager sets of size continuum. Arch. Math. Logic, 42(8):769779, 17
2003. 18
[BS10] Tomek Bartoszynski and Saharon Shelah. Dual Borel conjecture and Cohen reals. J. Symbolic Logic, 75(4):12931310, 19
2010. 20
[Car93] Timothy J. Carlson. Strong measure zero and strongly meager sets. Proc. Amer. Math. Soc., 118(2):577586, 1993.
21
[GK06] Martin Goldstern and Jakob Kellner. New reals: can live with them, can live without them. MLQ Math. Log. Q., 52(2):115 22
124, 2006.
23
[GMS73] Fred Galvin, Jan Mycielski, and Robert M. Solovay. Strong measure zero sets. Notices of the AMS, pages A280, 1973.
24
[Gol93] Martin Goldstern. Tools for your forcing construction. In Set theory of the reals (Ramat Gan, 1991), volume 6 of Israel 25
Math. Conf. Proc., pages 305360. Bar-Ilan Univ., Ramat Gan, 1993.
26
[Jec03] Thomas Jech. Set theory. Springer Monographs in Mathematics. Springer-Verlag, Berlin, 2003. The third millennium 27
edition, revised and expanded.
28
[JS90] Haim Judah and Saharon Shelah. The Kunen-Miller chart (Lebesgue measure, the Baire property, Laver reals and preser- 29
vation theorems for forcing). J. Symbolic Logic, 55(3):909927, 1990.
30
[Kel] Jakob Kellner. Non-elementary proper forcing. preprint, http://arxiv.org/abs/0910.2132.
31
[KS05] Jakob Kellner and Saharon Shelah. Preserving preservation. J. Symbolic Logic, 70(3):914945, 2005.
32
[Lav76] Richard Laver. On the consistency of Borels conjecture. Acta Math., 137(3-4):151169, 1976.
33
[Paw96a] Janusz Pawlikowski. A characterization of strong measure zero sets. Israel J. Math., 93:171183, 1996.
34
[Paw96b] Janusz Pawlikowski. Lavers forcing and outer measure. In Set theory (Boise, ID, 19921994), volume 192 of Contemp. 35
Math., pages 7176. Amer. Math. Soc., Providence, RI, 1996.
36
[She98] Saharon Shelah. Proper and improper forcing. Perspectives in Mathematical Logic. Springer-Verlag, Berlin, second edi- 37
tion, 1998.
38
[She04] S. Shelah. Properness without elementaricity. J. Appl. Anal., 10(2):169289, 2004.
39
[She06] S. Shelah. Non-Cohen oracle C.C.C. J. Appl. Anal., 12(1):117, 2006.
40
[She10] Saharon Shelah. Large continuum, oracles. Cent. Eur. J. Math., 8(2):213234, 2010.
41
[Sie28] W. Sierpi nski. Sur un ensemble non d enombrable, dont toute image continue est de mesure nulle. Fund. Math., 11:302 42
304, 1928.
43
Ixsrrrtr r ta Drskarrr Mxrnrmxrrk txo Gromrrarr, Trcnxrscnr Uxrvrasrr xr Wrrx, Wrroxra Hxtrrsraxsr 810/104, 1040 Wrrx, 44
Atsrarx 45
E-mail address: martin.goldstern@tuwien.ac.at
46
URL: http://www.tuwien.ac.at/goldstern/
47
Ktar G oori Rrsrxacn Crxrra roa Mxrnrmxrrcxi Loorc, Uxrvrasrr xr Wrrx, W xnarxora Sraxsr 25, 1090 Wrrx, Atsrarx
48
E-mail address: kellner@fsmat.at 49
URL: http://www.logic.univie.ac.at/kellner/
50
Erxsrrrx Ixsrrrtrr or Mxrnrmxrrcs, Eomoxo J. Sxrax Cxmrts, Grvxr Rxm, Tnr Hraarw Uxrvrasrr. or Jratsxirm, Jratsxirm, 51
91904, Isaxri, xxo Drrxarmrxr or Mxrnrmxrrcs, Rtroras Uxrvrasrr., Nrw Batxswrck, NJ 08854, USA
52
E-mail address: shelah@math.huji.ac.il
53
URL: http://shelah.logic.at/
54
Ixsrrrtr r ta Drskarrr Mxrnrmxrrk txo Gromrrarr, Trcnxrscnr Uxrvrasrr xr Wrrx, Wrroxra Hxtrrsraxsr 810/104, 1040 Wrrx, 55
Atsrarx 1
9
6
9


r
e
v
i
s
i
o
n
:
2
0
1
1
-
0
5
-
2
8







m
o
d
i
f
i
e
d
:
2
0
1
1
-
0
5
-
2
9


BOREL CONJECTURE AND DUAL BOREL CONJECTURE 49
E-mail address: wolfgang.wohofsky@gmx.at
2
URL: http://www.wohofsky.eu/math/
2033

You might also like